You are on page 1of 45

VERBAL ABILITY

HSSTS301_4210717

SESSION – 1

READING COMPREHENSION – I

Directions for Q1 to Q10: Read the following information and answer the questions given below.

The Most Productive Day of the Week


It's possible that while you are at work, you may dream about a month of Sundays, but your boss wishes for a week of
Tuesdays. That's because s/he probably knows that productivity is one of the main factors bolstering a company's
growth. And a recent poll shows that workers are most productive on Tuesdays! Accountemps, an employment
agency, conducted a national survey of office managers, which shows that by the middle of the week, they see a
dramatic productivity decrease. While Monday is considered second in "productivity value," only nine percent of
office managers think Wednesday is the peak productivity day. Five percent believe it is Thursday. And Friday, well,
you can just imagine! However, forty-eight percent of the managers polled said that Tuesday is, by far, the most
productive day of the week.
A close analysis of workweek rhythms would turn up some obvious reasons for those survey results. First of all,
Monday is overloaded with meetings, designed to "get things moving," and everybody knows meetings aren't very
productive. Wednesday is "hump day" - get over it as painlessly as possible, a worker thinks, and the week is more
than halfway over. On Thursday, people are running out of steam; and Friday, everybody's thinking about the
weekend. There are reasons why the other days aren't productive, but what makes Tuesday special? Tuesdays,
employees hit peak performance because they are very focused on day-to-day activities. Also, it's usually the first day
of the week when they're focused on their own task. They're not in meetings that take them away from their primary
responsibilities. Actually, Tuesdays can be quite hectic. Workers are arriving at work fairly frantic. And so, in 10 hours,
they're doing 20 hours work. That's productive, but it's also tough.
This does not mean that nothing happens on the last three days of the workweek. Things do not get so lax that people
are sitting with their feet on desks, sipping coffee and talking on the phone all day, but there's a definite lack of focus.
The pace softens and the rhythm slows down. And this is not healthy: it produces fatigue and lowers productivity. To
prevent this midweek slowdown, some management consultants suggest that employers avoid jamming so many
meetings into Mondays. Work deadlines can be rescheduled to stretch out the workflow. Variations in productivity are
only natural, but both workers and bosses win when the peaks and valleys are less dramatic than they are now.
1. This reading does not explain
(a) the concern managers have about low productivity on different week-days and how it affects their companies
(b) the reasons for differing productivity levels on different week-days
(c) the results of a survey on low productivity
2. According to this survey which day of the week should be the most productive?
(a) Monday (b) Thursday (c) Wednesday
3. According to the survey workers are most stressed on
(a) Monday (b) Tuesday (c) Thursday
4. The writer is suggesting that
(a) workers would be more productive if they worked on other days as they do on Tuesday
(b) workers would be more productive if they attended fewer meetings
(c) workers would be more productive if the work deadlines were readjusted
5. In general this article
(a) reports the surprising results from a national survey of office managers
(b) gives the reasons why Tuesday is a particularly productive day in the work week
(c) reports on the productivity for different work days and makes suggestions for making work week
productivity more balanced

Page 1 of 45
VERBAL ABILITY

HSSTS301_4210717

Okinawans Know the Formula for Longevity


Okinawa is a small island in Japan. Some of the people who live there are the poorest people in the country. But
Okinawans enjoy riches of a different kind—they have the longest life-expectancy rate in the world. The average
Okinawan woman lives to the age of 86, while the average Okinawan man lives to be 78. The island also has the
highest ratio of centenarians in the world. At the beginning of 2002, 457 Okinawans were 100 years old or older.
That's 34.7 centenarians per 100,000 Okinawans. Compare that to the United States, which only has 10 centenarians
per 100,000 Americans.
Okinawans don't wait out the final years of their long lives in hospitals or convalescent care facilities. They remain
healthy through their final days. Dementia, senility, and coronary heart disease are almost nonexistent among these
older Okinawans. In fact, compared to the United States, Okinawa's death rate for coronary heart disease is 80%
lower. And Okinawans are more likely than Americans to survive heart attacks on the rare occasions that heart
attacks occur.
So what is the secret to the Okinawans' health and longevity? Is it a healthy diet? Exercise? Stress reduction? Attention
to spirituality? It isn't any one of these things, but a balanced combination of the four.
The Okinawan diet consists mostly of vegetables and whole grains. Okinawans also eat a lot of fruit, soy products, like
tofu, and fish rich in Omega-3 fatty acids, such as tuna, mackerel, and salmon. Most Okinawans drink at least six
glasses of water and six cups of tea every day. The green tea that many of them drink is lower in caffeine than other
teas and is rich in antioxidants, which are believed to help prevent cancer, heart disease, and stroke.
Okinawans continue to practice martial arts, ride bicycles, dance, garden, walk, and fish well into their old age, which
helps keep them looking and feeling healthy. They have relaxed schedules, which reduces their stress levels. And they
pay attention to their spirituality and their inner selves through prayer and meditation. Each of these behaviors is an
important piece of the Okinawan formula for health and longevity.

There is evidence that it is their behavior, not genetics, that allows Okinawans to live longer than most of us.
Okinawans living in Brazil and eating a typical Brazilian diet rich in red meat have a life expectancy 17 years lower
than that of their countrymen in Okinawa. This means that there is hope for the rest of us to learn the habits that
Okinawans have developed so we can live long, healthy lives too.

6. Okinawans _____ than anyone else in the world.


(a) are richer (b) live longer (c) farm and fish more
7. People in Okinawa eat a lot of _____.
(a) vegetables, fruit, soy, and whole grains
(b) vegetables, read meat, soy, and fish
(c) vegetables, soy, milk products, and whole grains
8. The four things that contribute to Okinawans' long lives are a healthy diet, exercise,
(a) longevity and spirituality
(b) low stress levels and spirituality
(c) martial arts and spirituality
9. At the beginning of 2002, there was an average of _____ centenarians per 100,000 people in Okinawa.
(a) 457 (b) 10 (c) 34.7

10. Okinawans have such long lives because of _____.


(a) genetics (b) their behavior (c) their diet

Page 2 of 45
VERBAL ABILITY

HSSTS301_4210717

SESSION – 1
READING COMPREHENSION PART – I
1. (a) the concern managers have about low productivity on different week-days and how it affects their
companies

2. (a) Monday

3. (a) Monday

4. (c) workers would be more productive if the work deadlines were readjusted.

5. (b) gives the reasons why Tuesday is a particularly productive day in the work week.

6. (b) live longer

7. (a) vegetables, fruit, soy, and whole grains

8. (b) low stress levels and spirituality

9. (c) 34.7

10. (b) their behavior

SESSION – 2
READING COMPREHENSION PART – II
Directions for Q1 to Q10: Read the following information and answer the questions given below.

Modern Day Meditation


In a world where bad news has become everyday news, people are turning to an ancient technique to deal with stress:
meditation. At meditation centers, prayer groups and yoga studios around the United States, more and more are
finding peace of mind by being quiet. Some use meditation to help deal with life changes; others, to process the painful
reality of political and social unrest around the world of the type that has been experienced more recently. Stress from
the September 11 terrorist attacks is "probably about 70 percent" of the reason one Chicago man started meditating
and practicing yoga with his new wife. He became so emotionally affected that he realized he needed help in managing
his stress. The yoga classes he takes begin and end with meditation. This "quiet time" helps him feel a lot more relaxed
and gives him more breath control. The fact is, though, that he is not alone.

Across the country, many are turning to more meditative exercise as they seek both psychological and physiological
relief. In addition to helping people work out their stress, these classes bring people together, in the same way that
religious services or other community activities have done in the past. Different schools of meditation teach particular
techniques, but they share a common basis - focusing attention on something your mind can return to if you are
distracted. This may be the rhythm of breathing, a mantra (a word or phrase repeated continually, either silently or
aloud), an object such as a candle flame or religious icon, a positive affirmation, feelings of loving kindness, or a
repetitive movement, as in walking or t'ai chi. Regardless of the specific technique or mode that is followed,
meditation has well-documented benefits. Medical research indicates that it causes a sharp decrease in metabolic
activity, reduced muscle tension, slower breathing, and a shift from faster beta brainwaves to slower alpha, theta and
delta waves. It also reduces high blood pressure. Practitioners are convinced that meditation is good for health
because it relaxes the body.

Page 3 of 45
VERBAL ABILITY

HSSTS301_4210717

For ages, meditation has been a core practice of many groups meeting in their communal or religious centers.
However, let's not forget that this is the twenty-first century. So, for those people who are too shy or busy to go to the
nearest meditation center, there are Internet sites that offer online guided meditation. One has a variety of
meditations from various religious traditions. At another, Jesuit priests post meditations and readings from the
Scriptures every day, and at still another, Buddhist and Hindu practitioners include music and visuals to accompany
their offerings. These websites allow anyone with a computer access to meditation at any time. The fact is that
whether online, at yoga classes, or at local spiritual centers, more people are turning to the practice of meditation.

1. According to this article meditation is becoming more popular today because


(a) people have to work much harder than in the past
(b) people have to work much harder to find peace of mind
(c) people have to cope with more difficulties in their personal and communal lives

2. The example of one Chicago man shows


(a) the physical and psychological benefits of yoga and meditation
(b) the physical benefits of yoga and meditation
(c) the psychological benefits of yoga and meditation

3. What is the basic underlying approach of all meditative techniques?


(a) People repeat a phrase over and over.
(b) People focus on something that they can return to.
(c) People concentrate on a candle flame or a religious object.

4. What two important points are explained in this reading?


(a) An explanation of the various teachers and the different schools of meditation.
(b) A description of the places to go and procedures people can follow to learn meditation.
(c) An explanation of the best practices and various health benefits from meditation.

5. What is one idea that is explained in this reading?


(a) The future growth of religious centers that teach meditation.
(b) The present day appeal of meditation classes compared to the past attraction to religious or community
celebrations.
(c) The increasing popularity of meditation through the availability of on-line resources.

The Second "Battle of New Orleans"


In 1815, the pirate-turned-patriot Jean Lafitte joined the future president Andrew Jackson to protect Louisiana, the
Mississippi River, and America from a British military invasion. Lafitte pulled, pushed, and floated his large guns
through bayous, swamps, and marshes to fight in the Battle of New Orleans. In a shocking victory, Jackson and Lafitte
routed the British and saved New Orleans, Louisiana and the Mississippi River for the United States. Today, due to the
work of modern man and progress, the wetlands that Lafitte crossed to protect his homeland are being swallowed by
the salt water of the Gulf of Mexico. The state of Louisiana and the Mississippi Delta are gradually disappearing. The
coastal wetlands are lost at the rate of 100 yards, the length of a football field, every 15-20 minutes.

Over many thousands of years, nature created Louisiana and the Mississippi Delta. Along the over 2,000-mile course,
the Mississippi River drains soil from much of America's heartland to create the rich land of the Mississippi Delta.
Over time, the Delta region grew abundant in vegetation, fish, ancient trees, and other life forms. But in the last two
centuries, the wetlands created by the Mississippi were drained for a variety of reasons. At first it was to stop diseases
that endangered people who moved there and to grow crops to feed America and the world. Then the valuable
cypress trees were cut down to build homes and to export the wood. Natural canals were turned into large channels to
facilitate transportation. This massive work was called "reclamation," the reclaiming of the land from the water that

Page 4 of 45
VERBAL ABILITY

HSSTS301_4210717

the Mississippi River delivered. Later, more land was reclaimed to develop a lucrative and thriving petroleum
industry. Then, more canals were needed to make way for large ships. Levees were built for protection from the
annual spring floods. Ultimately, less fresh water drained into the Mississippi Delta.

Now, the natural habitat that nurtured wildlife, 300 species of birds, thousands of species of fish, flora, and other
ecological systems is rapidly disappearing. The ecology-friendly island barriers that were created by the flow of the
Mississippi and protected the coast have crumbled into the salt water. The salt water, without the island barriers and
the soil drainage of the Mississippi, now comes to meet the great river. The fresh water ecological system is dying and
disappearing. Daily, the salty Gulf waters cover what was once a thriving wetland full of fresh water life.

There are plans to save the coast, re-establish freshwater flooding, and drive back the advancing salt water. One plan
spends $14 billion dollars over the next 20 years to save this fragile, endangered, and disappearing land. Most people
see a need to protect their homeland. Some others disagree with taking immediate steps to protect the wetlands and
environment. It is safe to say that the battle has begun. Whether an Andrew Jackson or a band of pirates led by Jean
Lafitte will come to Louisiana's rescue in this war for the environment is not yet known. We can be sure though, in the
end, nature will have the final say.

6. Andrew Jackson ____________ in 1815.


(a) protected Louisiana from the Mississippi River
(b) was forced to join the British to fight pirates on the Mississippi
(c) attempted to stop a British invasion of America
7. In Louisiana, coastal wetlands ______________.
(a) are disappearing daily
(b) enjoy the refreshing salt water from the Gulf
(c) are vital to the expansion of the state of Louisiana
8. The Mississippi River _______________.
(a) has remained the way it has always been for thousands of years
(b) has changed rapidly in the last few years to meet the needs of a growing population
(c) has changed steadily over the last two centuries to meet the needs of the population
9. The island barriers _____________.
(a) have grown as a result of the flow of the Mississippi
(b) have changed the flow of the Mississippi
(c) have disappeared as a result of the flow of the Mississippi

10. There is _______________ to drive back the salt water and save the wetlands.
(a) general agreement that a plan should be made
(b) complete agreement that a plan must be made
(c) little agreement that a plan can be made

SESSION – 2
READING COMPREHENSION PART – II

1. (c) people have to cope with more difficulties in their personal and communal lives.

2. (a) the physical and psychological benefits of yoga and meditation.

3. (b) people focus on something that they can return to.

4. (b) A description of the places to go and procedures people can follow to learn meditation.

5. (c) The increasing popularity of meditation through the availability of on-line resources.

Page 5 of 45
VERBAL ABILITY

HSSTS301_4210717

6. (c) attempted to stop a British invasion of America

7. (a) are disappearing daily

8. (c) has changed steadily over the last two centuries to meet the needs of the population.

9. (c) have disappeared as a result of the flow of the Mississippi.

10. (a) general agreement that a plan should be made

SESSION – 3

PARA-JUMBLES - I

1. What are para jumbles?


Para jumbles are, as the name suggests, jumbled paragraphs. You are expected to rearrange the sentences so that they
make logical sense.

Solving para jumbles involves a strong grasp of context, language, grammar and more importantly, it requires
common sense.

2. Question Types
Most para jumble questions are of two types:
1. Moving jumbles: This is the plain vanilla version where a bunch of sentences are jumbled and given to you. And
the answer is a certain combination of these sentences.
2. Anchor jumbles: In an anchor jumble, the first and the last sentence are given. And the sentences in between
are in random order. The answer is again, a specific combination of these sentences.

3. Approach Methodology
The different methodologies that can be adopted for solving para jumbles are:
(a) Acronym Approach - full form vs. short form
(b) Time Sequence Approach – TSA - either dates or time sequence indicating words
(c) Examples Approach – EA - after an hypothesis or theory
(d) Articles - definite and indefinite
(e) Noun, Pronoun, and Demonstrative Adjective - NPDA Approach - limited to not just noun
(f) Opening - Closing Sentence Approach - OCSA - supported or free, general or need previous explanation
(g) Key Words Approach - KWA - words repeated in two consecutive sentences
(h) Look for the Conjunctions that play a very important role in joining two or more sentences. Such as
after, although, as far as, as long as, as soon as, because, before, if, since, so, though, unless, until, when,
whenever, whereas etc.

4. What to see while doing para jumbles


 Even the toughest para jumbles have two or three sentences that form a link. Try to spot a link that has to
go together and then eliminate the options that do not have that link. This is the first round of elimination.
 The coordinating conjunctions are also very important. Coordinating conjunctions are: for, and, nor, but,
or, yet and so.
 Keep an eye on Adverbs like even, also, still, however, nevertheless, notwithstanding etc. Such adverbs
help link two or more sentences.

Page 6 of 45
VERBAL ABILITY

HSSTS301_4210717

 Use the pronouns to your advantage. Ideally a pronoun in one sentence should always point to a noun in
the sentence immediately preceding it.
 General information always precedes specific information. That is the thumb rule. Information/idea that is
more universal should come first followed by information/idea that is less universal or more specific.
 Information of an idea/entity comes first followed by general description. Tangible descriptions should
always take precedence over intangible descriptions. If you describe a man’s personality, describe his
physical features and then move to his intangible aspects, same rule applying to other things.
 The conclusion as a rule, always comes at the end. Any sentence that follows a conclusion creates a break
in the flow of ideas, a feeling of this awkwardness should help you in finding the last sentence of the
paragraph.
 Keep an eye on the chronology and the importance of events. An event of lesser significance comes later
but an event of greater significance always comes first.
 Lastly never rush with your answers. Be patient. Mark the answer only after you have confidently evaluated
the other options.

5. Establishing link between two sentences and then examining the options

Suppose you establish the link 'BA'. The given options are:
(a) DABC (b) ACDB (c) CBAD (d) DBAC.
Now you are left with option (c) and (d) to examine. You read the sentences in the order given by these two options
and use your methods again to determine which one is correct.
Is establishing links between two sentences easy?

Directions for Q1 to Q20: Each question has a number of sentences which, when properly sequenced, form a
coherent paragraph. Each sentence is labeled with a letter. Choose the most logical order of sentences from among the
given choices to construct a coherent paragraph.

1. A. Tigers have been known to eat up to 60 pounds (27 kilograms) of meat in one night, but more often they
consume about 12 pounds (5 kilograms) during a meal.
B. It may take days for a tiger to finish eating its kill. The cat eats until it's full, and then covers the carcass with
leaves and dirt.
C. When it's hungry again, the tiger comes back to feed some more, until the meat is gone.
(a) ABC (b) BAC (c) BCA (d) None of these

2. A. Reasons include demand for tiger fur as well as for other body parts that many people use in traditional
medicines.
B. Tigers' habitat has also dwindled seriously as humans have developed land for uses such as farming and logging.
C. The whole tiger species is endangered throughout its range.
(a) ABC (b) BAC (c) CAB (d) None of these

3. A. Connie St Louis, who wrote the original account of Sir Tim’s speech which prompted the row that led him to
resign from his posts at University College London and the Royal Society, told The London Evening Standard
she has received “unpleasant” emails and tweets from his supporters.
B. The journalist who accused Nobel Prize-winning scientist Sir Tim Hunt of sexism said she has been the
victim of Twitter trolls and accused the “establishment” of “bullying” and “protecting their own”.
C. Ms St Louis, a lecturer in science journalism at City University, added: “It doesn’t make me change my mind.
I am standing by my story.”
(a) ABC (b) CAB (c) BCA (d) BAC

Page 7 of 45
VERBAL ABILITY

HSSTS301_4210717

4. A. Hamlet is an enigma.
B. His challenge to Guildenstern rings true for everyone who seeks to know him: "You would pluck out the
heart of my mystery."
C. None of us ever really does.
D. No matter how many ways critics examine him, no absolute truth emerges.
(a) ABCD (b) ADBC (c) BCAD (d) None of these
5. A. The results, presented on Tuesday at the Alzheimer’s Society annual research conference in Manchester,
have been hailed as “hugely promising” because they involve medicines that are already known to be safe
and well-tolerated in people – potentially cutting years from the timeline for drugs to reach patients.
B. Speaking ahead of her presentation, Giovanna Mallucci, professor of clinical neuroscience at the University
of Cambridge, said: “It’s really exciting. They’re licensed drugs.
C. This means you’d do a straightforward basic clinical trial on a small group of patients because these are not
new compounds, they’re known drugs.”
D. Two licensed drugs have been shown to halt brain degeneration in mice, raising the prospect of a rapid
acceleration in the search for a medicine to beat Alzheimer’s disease.
(a) ACBD (b) BDAC (c) CDAB (d) DABC
6. A. The latest research suggests that the build-up of abnormal amyloid proteins seen in Alzheimer’s,
Parkinson’s and CJD are simply a visible symptom of neuro-degeneration, but not the root cause.
B. Instead, Prof Mallucci argues, the damage really occurs when a natural defence mechanism in the brain
responds to the accumulation of plaques by switching off the production of all new proteins, wrecking the
brain’s ability to carry out essential repairs.
C. In mice with prion disease, a neurodegenerative disorder that scientists use as a model for dementia in
mice, both treatments were shown to restore protein production, stop brain cells from dying off and prevent
memory loss.
D. The drugs work by inhibiting an enzyme, called Perk that activates the defence mechanism.
(a) ABCD (b) BACD (c) DCBA (d) CABD

7. A. Now, you can get a keyring-friendly mini-pack, meaning that it can be on hand at all times.
B. It may be derided as a modish condiment, but it is extremely tasty, and sparks the kind of devotion that prompts
fans of the bright red bottle to buy T-shirts with it on the front and say things like: “Oh, I have it with everything.”
C. So, I decided to spend a day having it with everything.
D. Sriracha hot sauce, which originated in eastern Thailand, is a mix of garlic, sugar, chillis, vinegar and salt.
(a) ABCD (b) DBAC (c) CADB (d) BDAC

8. A. And dropped out after the first year.


B. Deep down I knew that what I really wanted to do was to write stories.
C. When I was growing up in Nigeria I was expected, as every student who did well was expected, to become a
doctor.
D. But I did what I was supposed to do and I went into medical school.
(a) CBDA (b) ACBD (c) DCBA (d) BCDA

9. A. This is because at 11 p.m. on Tuesday night Greece failed to make a €1.6bn payment due to the International
Monetary Fund, one of the country’s creditors, thus becoming the first developed nation to go into “arrears”
with the fund.
B. The effective sovereign default has ratcheted up pressure on the ECB to further restrict the funding
available to banks, which have been swapping Greek government bonds for funding from the Frankfurt-
based central bank.

Page 8 of 45
VERBAL ABILITY

HSSTS301_4210717

C. Greece’s banks have been shut since 29 June, after Mr Tsipras’ surprise decision to hold a referendum
prompted the European Central Bank (ECB) to limit the emergency lending available.
D. The ECB decided to maintain the current funding limit.
(a) CABD (b) BADC (c) DCBA (d) CBDA

10. A. "My opinion is that we need to invest for something to grow, whether that is a person, a company or a
nation like Greece," Feeney explained to i100.co.uk.
B. His reasoning: there are 503 million people in the EU. We would only need to chip in about €3 each (the
same price as a feta and olive salad) to raise the money.
C. With Greece's government and its European creditors hitting loggerheads over the impending deadline for
its debt repayments, one 29-year-old from Yorkshire has come up with a novel solution.
D. Thom Feeney, who says he is fed up of "dithering" politicians, has set up a crowd funding page on IndieGoGo
where he hopes to raise €1.6bn (£1.14bn) to help bail out the country.
(a) ABCD (b) CDBA (c) ACBD (d) DCBA

11. A. A story not only allows us to understand ourselves and how we live, it can also bring about change.
B. Forced Entertainment’s retelling of the entire Shakespearean canon using household objects, from salt and
pepper cellars to bottles of olive oil, is entirely serious.
C. One is simply the bare bones, but a story is something more; its meanings depend on the storyteller and
how the tale is delivered, which parts are emphasised and which downplayed.
D. As the Mahabharata tells us at its beginning: “If you listen carefully, at the end you’ll be someone else.”
E. Not least because it points up the vast difference between telling a plot and telling a story.
(a) ACDBE (b) BECAD (c) DACBE (d) BEACD

12. A. But seen from abroad, the UK looks a haven of stability.


B. Consumers are spending and businesses are investing.
C. In the past, news of a deficit would have alarmed foreign holders of sterling.
D. They would have pulled their money out of the UK, prompting a fall in the value of the pound, which would
have helped boost exports and trim imports, leading to an improvement in the balance of payments.
E. Growth was stronger in the first quarter at 0.4% than previously believed.
(a) CDAEB (b) ABDCE (c) CADBE (d) BADCE

13. A. At the age of twenty-one he wrote a treatise upon the binomial theorem which has had a European vogue.
B. But the man had hereditary tendencies of the most diabolical kind.
C. Moriarty is a man of good birth and excellent education, endowed by nature with a phenomenal
mathematical faculty.
D. A criminal strain ran in his blood, which, instead of being modified, was increased and rendered infinitely
more dangerous by his extraordinary mental powers.
E. On the strength of it, he won the mathematical chair at one of our smaller universities, and had, to all
appearances, a most brilliant career before him.
(a) ACDEB (b) ABCED (c) CAEBC (d) CEBAD

14. A. Since then, intelligence tests have been mostly used to separate dull children in school from average or
bright children, so that special education can be provided to the dull.
B. In other words, intelligence tests give us a norm for each age.
C. Intelligence is expressed as intelligence quotient and tests are developed to indicate what an average child
of a certain age can do-what a 5-year-old can answer, but a 4-year-old cannot, for instance.

Page 9 of 45
VERBAL ABILITY

HSSTS301_4210717

D. Binet developed the first set of such tests in the early 1900s to find out which children in school needed
special attention.
E. Intelligence can be measured by tests.
(a) CDABE (b) DECAB (c) EDACB (d) CBADE

15. A. If the market falls to reflect our notions of equity, government intervention may be needed to redistribute
income.
B. Beyond the question of what to produce, we are also concerned about for whom output is to be produced.
C. In general, the market mechanism tends to answer the basic question of for whom to produce by
distributing a larger share of total output to those with the most income.
D. Although this result may be efficient, it is not necessarily equitable.
E. Is the distribution of goods and services generated by the marketplace “fair”?
(a) BEACD (b) DCBEA (c) CBEAD (d) DACBE

16. A. I wanted to be taken seriously.


B. Not about the teaching material, because I was well prepared and I was teaching what I enjoyed.
C. The first time I taught a writing class in graduate school, I was worried.
D. I really wanted to wear my shiny lip gloss and my girly skirt, but I decided not to.
E. Instead I was worried about what to wear.
F. And I was worried that if I looked too feminine, I would not be taken seriously.
(a) ABDCEF (b) CBFDAE (c) CBEADF (d) ABCFED

17. A. One was in The Famous Victories of Henry V, created for the RSC’s First Encounters programme, which
cleverly takes a knife to both text and plot in a significantly pared-down version of both parts of Henry IV,
plus Henry V.
B. And not a single one of them was exactly as it appears on the page.
C. The second was near the beginning of Ivo van Hove’s Kings of War, performed in Dutch, which condenses
several of Shakespeare’s plays to explore the nature of kingship and the responsibilities of leaders.
D. The last was on Sunday night, watching some of Forced Entertainment’s table-top Shakespeare live-
streamed on the Guardian’s website.
E. Were any of these performances less Shakespearean than a full, uncut performance of the original text in
English?
F. Over the last couple of weeks I’ve seen three very different versions of the scene in Henry IV Part II, in which
the future Henry V tries on his ailing father’s crown.
G. I’d say a resounding no.
(a) ACBDEFG (b) GFEDACB (c) FBACDEG (d) DEGACBF

18. A. A bowl of hummus weighing 4,090kg.


B. The Lebanese fought back, making 10,452kg worth of hummus.
C. Its response?
D. As a Palestinian citizen of Israel, he takes his food history seriously, as do most residents of Abu Ghosh.
E. That’s why they were so affronted when in 2009 Lebanon unilaterally declared hummus Lebanese and
made a bowl of it weighing 2,056kg to prove it.
F. The chef’s hummus has been voted by the Jewish Journal’s food blog as the best in Israel and his village once
held the Guinness world record for the biggest bowl of hummus.
G. The village of Abu Ghosh wasn’t having it.
(a) GBCDEAF (b) FDEGCAB (c) AFBGDCE (d) ABCDEFG

Page 10 of 45
VERBAL ABILITY

HSSTS301_4210717

19. A. Although there are large regional variations, it is not infrequent to find a large number of people sitting here
and there and doing nothing.
B. Once in office, they receive friends and relatives who feel free to call any time without prior appointment.
C. While working, one is struck by the slow and clumsy actions and reactions, indifferent attitudes, procedure
rather than outcome orientation, and the lack of consideration for others.
D. Even those who are employed often come late to the office and leave early unless they are forced to be
punctual.
E. Work is not intrinsically valued in India)
F. Quite often people visit ailing friends and relatives or go out of their way to help them in their personal
matters even during office hours.
(a) ECADBF (b) EADCFB (c) EADBFC (d) ABFCBE (e) None of these

20. A. It wasn’t reviewed by any of the national newspapers or tipped to be a Christmas chart topper.
B. But Chimamanda Ngozi Adichie’s We Should All Be Feminists might just be the most important book you
read all year.
C. Based on a TEDx talk the Nigerian writer gave in 2012, We Should All Be Feminists is a potent tour de force
on the subject of gender equality; one that whispers rather than shouts and confides rather than chides.
D. There is no excuse not to read it – particularly as it is just 52 pages long.
E. A lunch break or commute would provide ample opportunity.
F. Earlier this month, a short essay by a bestselling, prize-winning author was published with little fanfare.
(a) FBDCAE (b) FADCBA (c) FADCBE (d) FABCDE

ANSWER KEY
1. Ans: [a] 2. Ans: [c] 3. Ans: [d] 4. Ans: [b] 5. Ans: [d]

6. Ans: [a] 7. Ans: [b] 8. Ans: [a] 9. Ans: [a] 10. Ans: [b]

11. Ans: [d] 12. Ans: [a] 13. Ans: [c] 14. Ans: [c] 15. Ans: [a]

16. Ans: [c] 17. Ans: [c] 18. Ans: [b] 19. Ans: [c] 20. Ans: [d]

SESSION – 4

PARA-JUMBLES – II

Directions for Q1 to Q13: Sentences given in each question, when properly sequenced, form a coherent paragraph. The
first and last sentences are 1 and 6, and the four in between are labeled A, B, C and D. Choose the most logical order of
these four sentences from among the five given choices to construct a coherent paragraph from sentences 1 to 6.

1. 1. What does the state do in a country where tax morality is very low?
A. It tries to spy upon the taxpayers.
B. It investigates income sources and spending patterns.
C. Exactly what the tax authority tries to do now even if inconsistently.
D. It could also encourage people to denounce to the tax authorities any conspicuously prosperous neighbours
why may be suspected of net paying their taxes properly.
6. The ultimate solution would be an Orwellian System.
(a) BACD (b) DBAC (c) ABCD (d) DCBA

Page 11 of 45
VERBAL ABILITY

HSSTS301_4210717

2. 1. The fragile Yugoslav State has uncertain future.


A. Thus there will surely be chaos and uncertainly if the people fail to settle their differences.
B. Sharp ideological differences already exist in the country.
C. Ethnic, regional, linguistic and material disparities are profound.
D. The country will also lose the excellent reputation it enjoyed in the international arena.
6. At worst, it will once more become vulnerable to international conspiracy and intrigue.
(a) BCAD (b) ADCB (c) ACBD (d) DBCA

3. 1. India’s experience of industrialization is characteristic of the difficulties faced by a newlyindependent


developing country.
A. In 1947 India was undoubtedly as underdeveloped country with one of the lowest per capita incomes in the
world.
B. Indian industrialization was the result of a conscious deliberate policy of growth by an indigenous political
elite.
C. Today India ranks fifth in the international community of nations if measured in terms of purchasing power.
D. Even today, however, the benefits of Indian industrialization since independence have not reached the
masses.
6. In India has been a limited success; one more example of growth without development.
(a) CDAB (b) DCBA (c) CABD (d) BACD

4. 1. The New Economic Policy comprises the various policy measures and changes introduced since July 1991.
A. There is a common thread running through all these measures.
B. The objective is simple – to improve the efficiency of the system.
C. The regulator mechanism involving multitude of controls has fragmented the capacity and reduced
competition even in the private sector.
D. The thrust of the new policy is towards creating a more competitive environment as a means to improving
the productivity and efficiency of the economy.
6. This is to be achieved by removing the barriers and restriction on the entry and growth of firms.
(a) DCAB (b) ABCD (c) BDAC (d) CDBA

5. 1. Commercial energy consumption shows an increasing trend and poses a major challenge for the future.
A. The demand for petroleum during 1996-97 and 2006-07 is anticipated to 81 million tons and 125 million
tons respectively.
B. According to the projections of the 14th power Survey Committee Report, the electricity generation
requirements from utilities will be about 415 billion units by 1996-97 and 824 billion units by 2006-07.
C. The production of coal should reach 303 million tons by 1996-97 to achieve plan targets and 460 million
tons by 2006-07.
D. The demand for petroleum products has already outstripped indigenous production.
6. Electricity is going to play a major role in the development of infrastructural facilities.
(a) DACB (b) CADB (c) BADC (d) ABCD

6. 1. Count Rumford is perhaps best known for his observations on the nature of heat.
A. He undertook several experiments in order to test the theories of the origin of frictional heat.
B. According to the colorists, the heat was produced by the ‘’ caloric’’ squeezed out of the chips in the process
of separating them from the larger pieces of metal.
C. Lavoisier had introduced the term ‘’caloric’’ for the weightless substance heat, and had included it among
the chemical elements along with carbon, nitrogen and oxygen.

Page 12 of 45
VERBAL ABILITY

HSSTS301_4210717

D. In the ammunitions factory in Munich, Rumford noticed that a considerable degree of heat developed in a
brass gun while it was being bored.
6. Rumford could not believe that the amount of heat generated could have come from the small amount of
dust created.
(a) ABCD (b) CBDA (c) ACDB (d) CDAB

7. 1. The death of cinema has been predicted annually.


A. It hasn’t happened.
B. It was said that the television would kill it off-and indeed audiences plummeted, reaching a low in 1984.
C. Film has enjoyed a renaissance, and audiences are nor roughly double of what they were a decade ago.
D. Then the home computer became the projected nemesis followed by satellite television.
6. Why? probably because even in the most atomized of societies, we human beings feel the need to share out
fantasies and our excitement.
(a) CADB (b) BDAC (c) ABDC (d) DABC

8. 1. Visual recognition involves storing and retrieving of memories.


A. Psychologists of the Gastalt school maintain that objects are recognized as a whole in a parallel procedure.
B. Neural activity, triggered by the eye, forms an image in the brain’s memory system that constitutes an
internal representation on the viewed object.
C. Controversy surrounds the question of whether recognition is a single one-step procedure or a serial step-
by-step one.
D. When an object is encountered again, it is matched with its internal recognition and thereby recognized.
6. The internal representation is matched with the retinal image in single operation.
(a) DBAC (b) DCAB (c) BDCA (d) CABD

9. 1. The history of mammals dates back at least to Triassic time.


A. Miocene and Pliocene time was marked by culmination of several groups and continued approach towards
modern characters.
B. Development was retarded, however until the sudden acceleration of evolutional change that occurred in
the oldest Paleocene.
C. In the Oligocene Epoch, there was further improvement, with appearance of some new lines and extinction
of theories.
D. This led in Eocene time to increase in average size, larger mental capacity, and special adaptations for
different modes of life.
6. The peak of the career of mammals in variety and average large size was attained in this epoch.
(a) BDCA (b) ACDB (c) BCDA (d) ACBD

10. 1. The wind had savage allies.


A. If it had not been for my closely fitted helmet, the explosions might have shattered my eardrums.
B. The first clap of thunder came as a deafening explosion that literally shook my teeth.
C. I did not hear the thunder I actually felt it – an almost unbearable physical experience.
D. I saw lightning all around me in every shape imaginable.
6. It was raining so torrentially that I thought I would drown in mid air.
(a) BCAD (b) CADB (c) CBDA (d) ACDB

11. 1. Managers need to differentiate among those who commit an error once, those who are repetitively errant
but can be corrected, and those who are basically wicked.
A. The persons in this category will resort to sweet-talk and make all sorts of promises on being caught, but, at
the first opportunity will revert to their bad ways.

Page 13 of 45
VERBAL ABILITY

HSSTS301_4210717

B. Managers must take ruthless action against the basically wicked and ensure their separation from the
organization at the earliest.
C. The first category needs to be corrected softly and duly counseled; the second category should be dealt with
firmly and duly counseled till they realize the danger of persisting with their errant behavior.
D. It is the last category of whom the managers must be most wary.
6. The punishment must be fair and based on the philosophy of giving all the possible opportunities and help
prior to taking ruthless action.
(a) ADCB (b) CDAB (c) CADB (d) BDAC

12. 1. Many space enthusiasts now warn that only private enterprise will truly drive human expansion into space,
and yet America’s government keeps ignoring them.
A. It may seem surprising, but there are large numbers of people who would spend hundreds of thousands of
dollars on a trip into space.
B. Tourism and entertainment are both possibilities.
C. Two people, so far, have spent $20 m, and another two are on their way.
D. What might cause market forces to take up the mission?
6. Film and television companies would also spend tens of millions if they could.
(a) DCAB (b) BACD (c) DBAC (d) ACBD

13. 1. Jane Austen died and came back as a fantasy writer.


A. The book itself has been called, by a media ever eager to summarise even 800-pages hardcover tomes into a
snappy catchphrase. “Harry Potter for adults”.
B. Unlike her previous avatar, the 21st century Ms Clarke (nee Austen) seems to be enjoying the attention
showered upon her and far from publishing her first book under a pseudonym, has been a central performer
at her own media circus.
C. The book, which she now calls her “debut”, began attracting media attention long before publication and on
release it’s been universally lauded.
D. In her new avatar, she calls herself Susanna Clarke, lives in Cambridge, and has authored a fat historical
fantasy novel set in the year 1860.
6. It’s also been praised by perhaps the best living author of British fantasy novels, Neil Gaiman, as
“unquestionably the finest English novel of the fantastic written in the past seventy years.”
(a) BADC (b) DBCA (c) DCAB (d) DCBA

Directions for Q14 to Q20: The first and the last parts of the sentence are marked 1 and 6. The rest of the sentence is
split into four parts and marked-P, Q, R and S. These four parts are not given in their proper order. Read the sentences
and find out which of the five combinations is correct.

14. 1. It depends upon the


P. a new heaven on earth or to destroy
Q. user, whether science will be used to create
R. the word in a
S. outlook and mentality of the
6. common conflagration.
(a) PQRS (b) SPQR (c) QPRS (d) SQPR

15. 1. The English


P. plundered the country
Q. and raw materials,

Page 14 of 45
VERBAL ABILITY

HSSTS301_4210717

R. as a result, the Company became rich


S. of precious mineral wealth
6. and the Indian people became poor.
(a) PQSR (b) SQPR (c) SRQP (d) PSQR

16. 1. The power and pride


P. for in the courage
Q. of Sparta was above all
R. discipline and skill of these troops
S. in its army,
6. it found its security and its ideal.
(a) SPRQ (b) PQRS (c) QSPR (d) PRQS

17. 1. It is the
P. the careful observer Q. apparently trivial phenomena
R. which gives even the S. intelligent eye of
6. their value.
(a) SPRQ (b) PQRS (c) QSPR (d) PRQS

18. 1. Nuclear test explosions


P. food as well as
Q. present and future generations
R. directly injuring the
S. take place, contaminating air and water and
6. of mankind.
(a) RQSP (b) PSQR (c) SPRQ (d) PQRS

19. 1. The club is an


P. nearby municipal school, for the children
Q. proper and healthy functioning of a
R. are constantly disturbed by its
S. intolerable nuisance to the
6. bizarre activities.
(a) QSRP (b) SQPR (c) PQRS (d) RPSQ

20. 1. What the country needs


P. and change tactics
Q. who would encourage players
R. are coaches and officials
S. to read the game as it progresses
6. accordingly.
(a) RQPS (b) SPRQ (c) QSPR (d) RQSP
1. Ans: [a] 2. Ans: [a] 3. Ans: [d] 4. Ans: [b] 5. Ans: [a]
6. Ans: [c] 7. Ans: [c] 8. Ans: [d] 9. Ans: [a] 10. Ans: [a]
11. Ans: [b] 12. Ans: [c] 13. Ans: [d] 14. Ans: [d] 15. Ans: [d]
16. Ans: [c] 17. Ans: [a] 18. Ans: [c] 19. Ans: [b] 20. Ans: [d]

Page 15 of 45
VERBAL ABILITY

HSSTS301_4210717

SESSION – 5

Slide 1: PREMISES AND CONCLUSIONS

Slide 2:

1. No one under eighteen-years-old can vote.

2. Harish is under eighteen-years-old.

3. Therefore, Harish cannot vote.

• Arguments are composed of one or more premises and a conclusion.

• Premises are statements offered as reasons for accepting another statement.

• A conclusion is a statement supported by reasons.

In this example, statements 1 and 2 are premises, and statement 3 is the conclusion.

In other words, statements 1 and 2 are the reasons leading to the conclusion in Statement 3.

Slide 3:

Here’s a graphical representation of Premise and Conclusion.

Slide 4: Chennai has a population of over 4 million. Mumbai has a population of close to 12 million. Delhi has a
population of almost 10 million.

The three sentences above do not constitute an argument because there is no conclusion, and arguments MUST have a
conclusion.

They are just a series of assertions and make no judgement of any kind.

This is the fundamental difference between ARGUMENTS and FACT SETS.

Slide 5: Differentiating premises from conclusions requires both practice and attention to the finer elements of
language.

Page 16 of 45
VERBAL ABILITY

HSSTS301_4210717

TIP #1: INDICATOR WORDS

Look for premise indicators--words like because, since, for, and given that--that provide clues when premises are being
offered.

Look for conclusion indicators--words like therefore, thus, hence, and so--that provide clues when conclusion indicators
are being offered.
Premise Indicators Conclusion Indicators

Since May be inferred from Therefore For this reason

Because May be derived from Hence For these reasons

For May be deduced from So It follows that

As Given that Accordingly I conclude that

Follows from In consequence Which shows that

As shown by Consequently Which means that

Inasmuch as Proves that Which entails that

As indicated by As a result Which implies that

The reason is that Thus We may infer

Slide 6 & 7: In a free society people have the right to take risks as long as they do not harm others as a result of taking
the risks. Therefore, it should be each person’s decision whether or not to wear a seat belt.

Identify the premise(s) and conclusion in this argument.

Premise: In a free society, people have the right to take risks as long as they do not harm others as a result of taking
the risks.

Conclusion: It should be each person’s decision whether or not to wear a seat belt.
Notice the word therefore in this passage. This is a premise indicator that gives us a clue.

Slide 8 & 9: Given that price of steel is rising, we will no longer be able to offer discounts on our car parts.
Identify the premise(s) and conclusion in this argument.
Premise: Price of steel is rising.

Conclusion: We will no longer be able to offer discounts on our car parts.


Notice the word given in the beginning. This is a premise indicator that gives us a clue.

Slide 10 & 11: Last year, the city of Indibad increased the size of its police force by fifty officers. This year, there was a
10 percent decrease in the number of violent crimes reported in Indibad. Clearly, a larger police force discourages
criminal activity.
Identify the premise(s) and conclusion in this argument.
Premise: Indibad increased the size of its police force and saw a decrease in reports of violent crime.
Conclusion: A larger police force discourages criminal activity.
Notice the word clearly in this passage. This tips us off that a premise is being offered.

Page 17 of 45
VERBAL ABILITY

HSSTS301_4210717

Slide 12 & 13: A good society treasures its dissidents and mavericks because it needs the creative thinking that
produces new hypotheses, expanded means, a larger set of alternatives, and, in general, the vigorous conversation
induced by fresh ideas.

Identify the premise(s) and conclusion in this argument.

Premise: A good society needs the creative thinking that produces new hypotheses, expanded means, a larger set of
alternatives, and, in general, the vigorous conversation induced by fresh ideas.

Conclusion: A good society treasures its dissidents and mavericks.

Notice the word because in this passage. This tips us off that a premise is being offered.

Slide 14 & 15: Television has a harmful effect on society. This can be seen from the poor school performance of
children who watch significant amounts of television and from the fact that children who watch more than six hours
of television in a day tend to read less than non-television watching children.

Identify the premise(s) and conclusion in this argument.

Premise 1: This can be seen from the poor school performance of children who watch significant amounts of television.

Premise 2: children who watch more than six hours of television in a day tend to read less than non-television
watching children.

Conclusion: Television has a harmful effect on society.

This argument features the premise indicator this can be seen from.

Slide 16 & 17: Make a will. Otherwise, the law will determine who gets your things.

Identify the premise(s) and conclusion in this argument.

Premise: If you don't make a will, the law will determine who gets your things.

Conclusion: You ought to make a will.

The word otherwise often functions--as it does here--as a premise indicator.

Notice that both the premise and the conclusion have been rephrased slightly. The premise has been rephrased in
order to make it a complete sentence. The conclusion has been restated in order to make clear that it is intended as a
statement rather than as a command.

Slide 18 & 19: Therefore, since higher debt has forced consumers to lower their savings, banks now have less money
to loan.

Identify the premise(s) and conclusion in this argument.

Premise: Higher debt has forced consumers to lower their savings.

Conclusion: Banks now have less money to loan.

Here, it could get confusing because the premise indicator since and the conclusion indicator therefore are placed next
to each other.

But you’ll see that therefore is still introducing the conclusion, but the appearance of the conclusion is interrupted by a
clause that contains the premise.

Clue: Notice the commas before since and at the end of the premise clause!

(Note: Similar confusing forms include “This, because…”, “Hence, due to…” etc.)

Page 18 of 45
VERBAL ABILITY

HSSTS301_4210717

Slide 20 & 21: Research universities also must aggressively support teaching. After all, a significant percentage of
their students are undergraduates, and such institutions are clearly obligated to provide them a quality education.

Identify the premise(s) and conclusion of this argument.

Premise 1: A significant percentage of research universities' students are undergraduates.

Premise 2: Such institutions are clearly obligated to provide undergraduates with a quality education.

Conclusion: Research universities also must aggressively support teaching.

Notice the phrase after all. This phrase is often used as a premise indicator.

Slide 22 & 23: Every trainer at the SMART teaches exactly one class per semester. SMART’s Professor Haridas,
therefore, is teaching exactly one class this semester. Moreover, I heard Professor Haridas say he was teaching only a
single class.

Identify the premise(s) and conclusion of this argument.

Premise : Every trainer at the SMART teaches exactly one class per semester.

Conclusion: Professor Haridas is teaching exactly one class this semester.

Therefore in the second sentence is a conclusion indicator.

But what about the third sentence?

It begins with the additional premise indicator moreover. The premise in this statement is non-essential to the
argument, but provides additional proof for the conclusion.

(Note: Other additional premise indicators include words like furthermore, besides, in addition, what’s more.)

Slide 24 & 25: Phenylketonurics are people who cannot metabolize the amino acid phenylalanine. There are dangers
associated with phenylketonurics, and products containing phenylalanine must carry a warning label that states,
“Phenylketonurics: contains phenylalanine”. In addition, all children in developed societies receive a phenylketonuria
test at birth. Hence, at the moment, we are doing as much as possible to protect against this condition.
Identify the premise(s) and conclusion of this argument.
Premise 1: Phenylketonurics are people who cannot metabolize the amino acid phenylalanine.
Premise 2: There are dangers associated with phenylketonurics, and products containing phenylalanine must carry a
warning label that states, “Phenylketonurics: contains phenylalanine”.
Additional Premise: In addition, all children in developed societies receive a phenylketonuria test at birth.
Conclusion: Hence, at the moment, we are doing as much as possible to protect against this condition.
Slide 26 & 27: The United States prison population is the world’s largest and consequently we must take steps to
reduce crime in this country. Although other countries have higher rates of incarceration, their statistics have no
bearing on the dilemma we face.
Identify the premise(s) and conclusion of this argument.
Premise: The United States prison population is the world’s largest.
Conclusion: We must take steps to reduce crime in this country.
The first sentence has a clear premise and conclusion. Consequently is a conclusion indicator.
But what about the second sentence?

Page 19 of 45
VERBAL ABILITY

HSSTS301_4210717

It offers a Counter Premise: Other countries’ higher rates of incarceration have no bearing on the dilemma we face.

The word although in the second sentence indicates a counter premise.

Slide 28: More on Counter Premises

Counter premises bring up points of opposition or comparison that NEGATE the author’s argument.

Common Counter Premise Indicators include

But, Yet, However, On the other hand, Admittedly, In contrast, Although, Even though, Still, Whereas, in spite of, Despite,
After all

While at first glance it may seem as if the author is opposing his own case, by raising the counter-premise and then
directly addressing the complaint, the author actually strengthens his case.

Slide 29 & 30: Wine is made by crushing grapes and eventually separating the juice from the grape skins. However,
the separated juice contains impurities and many wineries do not filter the juice. These wineries claim the unfiltered
juice ultimately provides a more flavorful and intense wine. Since these wine makers are experts, we should trust
their judgement and not shy away from unfiltered wine.

Identify the premise(s) and conclusion of this argument.

Premise 1: Wine is made by crushing grapes and eventually separating the juice from the grape skins.

Counter Premise: However, the separated juice contains impurities and many wineries do not filter the juice.

Premise 2: These wineries claim the unfiltered juice ultimately provides a more flavorful and intense wine.

Premise 3: These wine makers are experts.

Conclusion: We should trust their judgement and not shy away from unfiltered wine.

Slide 31 & 32: During last night’s robbery, the thief was unable to open the safe. Thus, last night’s robbery was
unsuccessful despite the fact that the thief stole several documents. After all, nothing in those documents was as
valuable as the money in the safe.

Identify the premise(s) and conclusion of this argument.

Premise: During last night’s robbery, the thief was unable to open the safe.

Counter Premise: Despite the fact that the thief stole several documents.

Additional Premise: After all, nothing in those documents was as valuable as the money in the safe.

Conclusion: Thus, last night’s robbery was unsuccessful.

Slide 33:

(a) Ask yourself, "What claim is the writer or speaker trying to prove?"

That claim will be the conclusion.

What if the passage has no indicator words?

TIP #2: THE WHY TEST

(b) Try putting the word "therefore" before each of the statements in turn. The statement it fits best will be the
conclusion.

Page 20 of 45
VERBAL ABILITY

HSSTS301_4210717

The Why Test. Once you have found the conclusion, ask yourself why the author believes the conclusion to be true.
The premises should provide the answer to the question. If you try the Why Test and the answer does not make sense,
you have probably reversed the conclusion and premises.

Slide 34 & 35: Companies have identified that giving workers the option of flexible lunch hours leads to happier
employees. Happier employees are more productive. Company X has flexible lunch hours. Company Z operates on a
strict 1 to 2 pm lunch break. Company X will certainly beat Company Z in worker productivity.

Identify the premise(s) and conclusion in this argument.

Premise 1: Flexible hours lead to happier employees, who will be more productive.

Premise 2: Company A has flexible hours and Company B does not.

Conclusion: Company A will certainly beat Company B in worker productivity.

There are no straightforward premise indicators here. However, it is possible to identify the conclusion by
substituting “therefore” before each sentence and confirming with the “Why Test”.

Slide 36 & 37: The Jews and Arabs have been fighting for centuries and I seriously doubt this will ever be resolved.
The United States should get out of this never-ending fight, or the next terrorist bomb might be in Washington--and it
just might be nuclear. (John G. Ferguson III, Letter to the Editor, USA Today, February 12, 2001)

Identify the premise(s) and conclusion in this argument.

Premise 1: The Jews and Arabs have been fighting for centuries.

Premise 2: There is serious doubt this will ever be resolved.

Premise 3: If the United States does not get out of this never-ending fight, the next terrorist bomb might be in
Washington--and it might be nuclear.

Conclusion: The United States should get out of the never-ending fight between the Jews and the Arabs.

In this passage, there are no indicator words to assist us, however the form of the last sentence ("X should do Y, or else
Z will happen") is a common conclusion-premise pattern.

Slide 38 & 39: The Indian census is not perfect: thousands of Indians probably go uncounted. However, the basic
statistical portrait of the nation painted by the census is accurate. Certainly some of the poor go uncounted,
particularly the homeless; but some of the rich go uncounted as well, because they are often abroad and travelling
between one residence and another.

Page 21 of 45
VERBAL ABILITY

HSSTS301_4210717

Identify the premise(s) and conclusion in this argument.

Counter Premise: The Indian census is not perfect: thousands of Indians probably go uncounted.

Premise: Some of the poor and some of the rich go uncounted.

Conclusion: The basic statistical portrait of the nation painted by the census is accurate.

(Note: Besides applying the ‘Why Test’, also remember: Whenever a sentence is followed by hard transition words like
however, but, yet, on the other hand, etc it cannot be the conclusion because the transition is a form of negation.)

SESSION – 6

Slide 1: Finding Assumptions


Slide 2: QUICK RECAP
All men are mortal.
Therefore, Haridas is mortal.
• Arguments are composed of one or more premises and a conclusion.
• Premises are statements offered as reasons for accepting another statement.
• A conclusion is a statement supported by reasons.
• Assumptions are the unstated link between premise and conclusion.

In this example, statements 1 is the premise, and statement 2 is the conclusion. However, there is a still a missing link
in the argument… i.e. Haridas is a man.

Slide 3:
Here’s another graphical representation of Assumptions.

(ASSUMPTIONS)

You can think of the conclusion of an argument as the top of a building, supported by the building itself (the premises)
and the unseen underground foundation (the assumptions).

Page 22 of 45
VERBAL ABILITY

HSSTS301_4210717

Slide 4 & 5

My dog won top prize at the dog show last year. Therefore, he will win again this year.

Identify the premise(s), assumption(s) and conclusion in this argument.

Premise: My dog won top prize at the dog show last year.

Conclusion: Therefore, he will win again this year.

Assumptions:

1. The author's dog will be entered in this year's show.

2. The author's dog will be eligible to win the top prize.

3. The author's dog will still be better than all of the other dogs from last year's show, and it will beat out any new
dogs who may be entered this year.

PPT Notes: Anyone of the above assumptions could serve as the correct answer, because each one is necessary in order
for the author to believe his or her conclusion. In other words, if the assumption in the correct answer were not true,
you could reject the conclusion on that basis alone. For example, if we learned that no dog is allowed to win the top
prize two years in a row, then clearly the author's dog is not going to win this year's prize.

At the same time, NONE of the assumptions listed is sufficient by itself to prove that the author's dog will definitely win
the prize this year. For instance, if the author's dog is eligible to win the prize this year, that does not mean it will
definitely do so. The correct answer does not need to make the conclusion definitely true. Indeed, most of the time, the
correct answer will only make the conclusion somewhat more likely to be true.

Slide 6

From the previous example, we learn that…


An argument can involve multiple assumptions. But what is most crucial, especially from the test perspective, is THE
ASSUMPTION on which the entire argument is built.
To identify the appropriate assumption, you should look for the assumption to:
(1) bridge a gap between any premise and the conclusion, and
(2) support/strengthen/validate the conclusion.
Slide 7 & 8
Identify the premise(s), assumption(s) and conclusion in this argument.
Studying regularly is one factor that has been shown to improve one's performance on the GMAT. Anita took the
GMAT and scored a 500. If she studies several times a week, Anita can expect to improve her score.
Premise 1: Studying regularly is one factor that has been shown to improve one's performance on the GMAT.
Premise 2: Anita took the GMAT and scored a 500.
Conclusion: If she studies several times a week, Anita can expect to improve her score.
Assumption: Studying several times a week qualifies as studying regularly.
Slide 9: ASSUMPTION vs. INFERENCE
An inference is a piece of information that can be logically deducted from the given set of statements.
Example: Everyone who has read this lesson will be more informed about Assumptions. Haridas has read this lesson.
Inference: Haridas is more informed about Assumptions.

Page 23 of 45
VERBAL ABILITY

HSSTS301_4210717

An assumption on the other hand is an unstated or “hidden” premise.

Example: Everyone who has read this lesson will be more informed about Assumptions. Haridas received this lesson
by email. Hence, Haridas is more informed about Assumptions.

Assumption: Haridas has read every email he received.

Slide 10 & 11: TYPES OF ASSUMPTIONS

Identify the assumption(s) in this argument.

Anil weighs less than 90 kilos. Therefore, he cannot have a successful career as a racecar driver.

In order to make the logical leap from Point A to Point B - that Anil is under 90 kilos and therefore cannot have a
successful career as a race car driver - we must insert an additional premise. This unstated premise is an assumption.

Assumption: In order to have a successful career as a racecar driver, one must weigh at least 90 kilos.

TYPE #1: Assumptions that fill a LOGIC GAP

These assumptions answer the question, "How do we logically get from Point A to Point B?"
In this case, P represents weighs less than 90 kilos and C represents cannot be a successful race car driver. An
assumption - call it A - sits between P and C. This A is the assumption that Anil cannot be a successful race car driver
unless he weighs 90 kilos or more. Note that if that assumption is not true, then the whole argument falls apart.

Slide 12 & 13: TYPES OF ASSUMPTIONS

Identify the assumption(s) in this argument.

Sidney's get-rich-quick scheme is sure to succeed. He will buy undervalued properties in foreclosure. Then he will
resell the properties to a local real estate developer and generate large profits.

Assumptions:

1. Undervalued properties exist in foreclosure and easily can be found and purchased by Sidney.

2. It also assumes that local real estate developers will want to buy such properties from Sidney and will be willing to
pay more than he paid in the first place.

TYPE #2: Assumptions that establish the FEASIBILITY of PREMISES in the argument

Page 24 of 45
VERBAL ABILITY

HSSTS301_4210717

In this case, P1 represents he will buy undervalued properties in foreclosure. P2 represents he will sell these
properties to a developer for large profits, and C represents Sidney's get-rich-quick scheme is sure to succeed.
Statement A makes explicit the assumption that Sidney actually CAN buy undervalued properties in foreclosure and
sell them for higher prices.
Slide 14 & 15: TYPES OF ASSUMPTIONS
Identify the assumption(s) in this argument.
A magazine published an article proclaiming that one can get a promotion by playing golf with one's boss. Kevin
concludes that the best way for him to get promoted is to take golf lessons and join his boss's country club.
These arguments will often use some type of superlative qualifier in the conclusion. That is, we are told that the given
path is the only way to reach a goal, or the best or worst way. In order for such a claim to be true, there cannot be
another way, or a better or worse way.
Assumption: There are no other better ways to gain a promotion than to play golf with one's boss.
TYPE #3: Assumptions that eliminate alternate paths to reach a given conclusion

In this case, X represents playing golf with one's boss and Z represents getting a promotion. The arrow indicates that X
is the best way to get to Z. Y makes explicit that there is no better way for Kevin to accomplish his goal.
There is one another assumption here… What is that?
Assumption 2: That Kevin’s boss plays golf! Now this is a TYPE #2 assumption.
PPT Notes: Notice that the conclusion uses the phrase the best way. This ignores the many other (probably more reliable)
ways that Kevin could get himself noticed and promoted. In other words, the argument assumes that there is no better
way for him to get promoted than to play golf with his boss.
Slide 16 & 17: TYPES OF ASSUMPTIONS
Identify the assumption(s) in this argument.
Scientists have discovered that people with Elmer's disease have elevated levels of elastomer in their blood. The
scientists concluded that a person without Elmer's disease but with elevated levels of elastomer in the blood is likely
to develop Elmer's disease in the future.
Assumption: Scientists are assuming that elevated levels of elastomer is the cause for Elmer’s disease.
TYPE #4: Assumptions that eliminate alternate causes for a given conclusion

Assuming high levels of elastomer is X and Elmer’s disease is Z, the argument is built on the certain assumption
that only X causes Z, and NOT that Z could cause X.
However, Elmer's disease and elastomer show up together and are correlated. But we do not know whether elevated
levels of elastomer causes Elmer's disease, or whether Elmer's disease causes elevated levels of elastomer. There is
even a third possibility: both Elmer's disease and elevated levels of elastomer could be caused by some third,
unknown factor.

Page 25 of 45
VERBAL ABILITY

HSSTS301_4210717

In short, correlation is not the same as causation.


PPT Notes: In short, if a premise provides a correlation between X and Z, and the conclusion provides a causality in one
direction (X causes Z), then the assumption you are usually looking for is that Z does NOT cause X. That is, you must rule
out the causality in the other direction.
Slide 18: PRACTICE QUESTION #1
When news periodicals begin forecasting a recession, people tend to spend less money on discretionary purchases.
Therefore, the perceived threat of recession decreases the willingness of people to purchase products that they regard
as optional or luxury goods.

The argument above assumes that ________.


(A) there are more luxury goods available after a recession is forecast.
(B) recently, the threat of recession has been increasingly publicized as news periodicals have grown more pervasive.
(C) most people do not regularly read news periodicals.
(D) people's perception of the threat of recession increases when news periodicals begin forecasting a recession.
(E) the people who spent the most money before a recession was forecast were among those who curtailed their
spending after the recession became apparent.

Answer: D
Type 1 Assumption  Fills a logic gap
Note that the premise states that the decreased spending is due to the fact that news periodicals are forecasting a
recession. Furthermore, the expressions perceived threat of recession (in the passage) and perception of the threat of
recession (in the answer choice) are essentially synonymous. Finally, the answer choice logically fills a gap between a
premise and the conclusion.
Premise: News periodicals publish the forecast of a recession
Assumption: As a result, people's perception of the threat of a recession increases
Premise: People spend less money on discretionary items
Conclusion: Therefore, the perceived threat causes people to spend less money

Slide 19: PRACTICE QUESTION #2

The local university recently hired a new soccer coach. Although she has several years' worth of coaching experience
and is a diligent student of the game, she was never a member of a collegiate soccer team. For this reason, the new
coach will be unable to build a successful program.
The argument above is based on which of the following assumptions?
(A) The local university should have hired a former collegiate soccer player as its new coach.
(B) Coaching experience is one of the most crucial factors for coaching success.
(C) The previous coach at the university was dismissed due to her lack of success.
(D) To build a successful soccer program as a coach, one must be a former collegiate soccer player.
(E) The university does not plan to provide the new coach with the resources necessary to build a successful
program.
Answer: D
Type 1 Assumption  Fills a logic gap

Slide 20: PRACTICE QUESTION #3

Methyltetrachloride (MTC) is a chemical found in some pesticides, glues. and sealants. Exposure to MTC can cause
people to develop asthma. In order to halve the nation's asthma rate, the government has announced that it will ban
all products containing MTC.

Page 26 of 45
VERBAL ABILITY

HSSTS301_4210717

The government's plan to halve the nation's asthma rate relies on which of the following assumptions?
(A) Exposure to MTC is responsible for no less than half of the nation's asthma cases.
(B) Products containing MTC are not necessary to the prosperity of the American economy.
(C) Asthma has reached epidemic proportions.
(D) Exercise and proper nutrition are helpful in maintaining respiratory health.
(E) Dust mites and pet fur cause asthma.

Answer: A
Type 1 Assumption  Fills a logic gap
The government's proposed ban on MTC cannot halve the nation's asthma rate unless MTC is actually the thing
responsible for at least half the nation's asthma cases. If other things are responsible for half (or more than half) of the
nation's asthma cases, then banning MTC will not have the desired effect of reducing asthma rates by half.

Slide 21: PRACTICE QUESTION #4

In order to achieve a substantial reduction in the extremely high number of car accidents in our country, we should
implement a radical new safe driving plan. Seat belts and air bags would be eliminated from the driver's side of all
automobiles to provide the driver with the strongest possible incentive to drive safely. Further, an electroshock system
would administer shocks to the driver if he or she exceeds the speed limit or engages in other unsafe driving practices.

In declaring that the radical new safe driving plan will help to reduce accident rates, the author assumes which of the
following?
(A) Many car accidents are caused, at least partially, by naturally occurring conditions such as rain and fog.
(B) Accidents in which one or both participants exceed 60 miles per hour account for a majority of all fatal car
accidents.
(C) A significant number of accidents are the result of negligence or other unsafe driving practices on the part of the
driver.
(D) To alleviate safety concerns, citizens should be allowed to reinstall their drivers' side seat belts and air bags at
their own expense.
(E) If not implemented properly, electroshock systems can cause heart attacks and other health problems.
Answer: C
Type 2 Assumption  Establishes Feasibility of Premise
The author assumes that a substantial number of accidents are due to driver error or could be avoided with a change in
driver. behavior. The author also assumes that the electroshock system will not itself cause a substantial number of
accidents, or health problems. that could lead to accidents.
Slide 22: PRACTICE QUESTION #5

Computers that can be identified and traced are of no use to thieves. So Cyber Infotech has instituted a plan to reduce
theft by etching the serial number of each computer both on the frame of the monitor and on the face of the hard drive
where it will be impossible to overlook.

Cyber Infotech’s plans assume which of the following?


(A) Cyber Infotech’s plan assumes that thieves do not steal computers that are of no value to them.
(B) Cyber Infotech’s competitors also plan to etch serial numbers on their computers.
(C) Thieves are more inclined to steal computers than any other type of office equipment.
(D) Etching the serial number on the computer has no effect on the performance of the computer.
(E) Imposing stiffer jail sentences for thieves would not decrease the number of computers stolen.
(F) USEFUL TECHNIQUE: The Reverse Test
One way to check whether an answer choice is a necessary assumption is to reverse or negate, that answer. If the
negation would weaken the argument, that answer is a necessary assumption.
Answer: A

Page 27 of 45
VERBAL ABILITY

HSSTS301_4210717

Slide 23: PRACTICE QUESTION #6


Editorial: The Intercontinental Bank should reallocate the voting shares of its members in order to more effectively
shape global economic policy. For example, China comprises about 15 percent of the world's gross domestic product
but has only a 3 percent voting share, whereas Belgium, with less than 1 percent of the global economy, has a 2
percent share.
Which of the following is an assumption upon which the author depends?
(A) The United States has a larger voting share of the Intercontinental Bank than does China or Belgium.
(B) The specific allocation of voting shares factors into the Intercontinental Bank's effectiveness in shaping global
economic policy.
(C) Only voting shares that are precisely proportional to each country's contribution to the global economy are
appropriate for the Intercontinental Bank.
(D) The Intercontinental Bank is necessary to the maintenance of a prosperous global economy.
(E) As one of the fastest growing economies, China should have a larger voting share in the Intercontinental Bank.
Answer: B
Type 2 Assumption  Establishes Feasibility of Premise
The argument does not actually present any evidence that the efficacy of the Intercontinental Bank will improve if Voting
shares are reallocated; this must be assumed in order to support the conclusion.
Slide 24: PRACTICE QUESTION #7
Advertisement: According to a recent research study, daily use of Fresh Start, a new toothpaste, reduces the risk of
developing dental cavities by over 20 percent. In addition, as a result of a new formula, the use of Fresh Start results in
whiter, healthier-looking teeth. Clearly, Fresh Start not only gives your teeth a beautiful look but also provides the
most reliable protection against dental cavities.
Which of the following is an assumption in the argument above?
(A) No other toothpaste provides more reliable protection against dental cavities.
(B) Fresh Start's formula does a better job of whitening teeth than do competitor formulas.
(C) People are just as interested in having beautiful teeth as they are in having healthy teeth.
(D) Fresh Start also prevents other dental disorders, such as gingivitis.
(E) Reliable protection against dental cavities, combined with excellent aesthetic properties, is likely to make Fresh
Start a popular toothpaste on the market.
Answer: A
Type 3 Assumption  Eliminates an Alternate Path to the same End
Since the argument singles out Fresh Start as the most reliable protection against dental cavities, the author must
assume that no other toothpaste provides stronger protection against cavities. Otherwise, the conclusion of the argument
is inaccurate.
Slide 25: PRACTICE QUESTION #8

Student Advisor: One of our exchange students faced multiple arguments with her parents over the course of the past
year. Not surprisingly, her grade point average (GPA) over the same period showed a steep decline. This is just one
example of a general truth: problematic family relationships can. cause significant academic difficulties for our students.

Which of the following is an assumption underlying the general truism claimed by the Student Advisor?
(A) Last year, the exchange student reduced the amount of time spent on academic work, resulting in a lower GPA.
(B) The decline in the GPA of the exchange student was not the reason for the student's arguments with her parents.
(C) School GPA is an accurate measure of a student's intellectual ability.
(D) If proper measures are not taken, the decline in the student's academic performance may become irreversible.
(E) Fluctuations in academic performance are typical for many students.

Page 28 of 45
VERBAL ABILITY

HSSTS301_4210717

Answer: B
Type 3 Assumption  Eliminates an Alternate Path to the same End
This assumption correctly eliminates the alternate model of causation, demonstrating that the decline in the GPA did not
cause the arguments between the student and her parents.

Slide 26: PRACTICE QUESTION #9

An industry analyst asserted in his recent report that the relative scarcity of housing in a particular market leads to
larger than normal increases in price. During the late 1990s, according to the analyst's report, occupancy rates -- a
measure of the percentage of housing occupied at a given time -- in crowded urban markets such as New York and San
Francisco hovered around 99.5%. During the same period, housing prices increased by as much as 100% per year,
compared to more normal past increases in the range of 5% to 15% per year.

Which of the following is an assumption that supports the analyst's assertion?


(A) In the housing market, there generally must be at least five buyers per seller in order to cause larger than normal
increases in price.
(B) Increases in demand often reflect an influx of new buyers into the marketplace or an unusual increase in buying
power on the part of the customer.
(C) The US housing market showed a larger than average increase in the 1990s across the country, not just in
crowded urban areas.
(D) Price increases do not cause people to withhold their houses from the market in the hopes that prices will
increase even further in the future.
(E) A significant rise in housing prices in a specific area may cause some potential buyers to relocate to other, less
pricey areas.

Answer: D
Type 4 Assumption Eliminates an Alternate Cause
The analyst claims that scarcity causes the price increase, but the reverse could also be true: the price increase could
cause the scarcity. Perhaps people wait to sell because they think the market will continue to rise, or perhaps people will
not sell their own property because they would then have to pay inflated prices for a new property. In order to conclude
that scarcity causes prices to rise, the analyst must assume that the "reverse" causation does not occur. Only Option D
denies reverse causation.

Slide 27: PRACTICE QUESTION #10

A group of college students participated in a sleep study where half of them slept for a full eight hours and the other
half were woken up two to three times during the night. The half that slept the full eight hours uninterrupted
performed better, on average, on a logical reasoning exam the next day than the other half whose sleep was
interrupted. Therefore, people have the best brain function when they sleep for a full eight hours, uninterrupted.

What are the three major flaws in the above argument?

This is an open ended question…THINK!

1. Sample of people: The people who were tested were all college students who, on average, are aged 18-22. However,
the conclusion is about people in general, which is a leap on logic.

2. Leap in Logic: A higher score on a logical reasoning test doesn’t necessarily mean a higher brain function. There are
lots of other parameters to consider.

3. The word best: The study discussed in the argument only tested two different sleep scenarios. Just because one is
better than the other, doesn’t necessarily mean that it is the best.

Page 29 of 45
VERBAL ABILITY

HSSTS301_4210717

SESSION – 7

CRITICAL REASONING
 Critical Reasoning questions involve reading brief arguments (each argument is generally one to three sentences
long) and answering questions relating to those arguments.
 In order to analyze arguments, it is important to understand their basic structure:
 Premises + (Assumptions) = Conclusion
 In words, premises and assumptions lead to a conclusion.
 PREMISES are STATED pieces of information or evidence that generally provide support for the given
conclusion.
 ASSUMPTIONS are UNSTATED parts of the argument that are NECESSARY to reach the given conclusion.
 The main point of the argument is the CONCLUSION, which is logically supported by the assumptions and
premises. Conclusions are in the form of an opinion or a claim.
 Here is an example of Critical Reasoning argument…

 There are two broad things we need to understand in order to answer Critical Reasoning questions effectively
and efficiently.

 We need to understand the specific information given for that question, and also how to conduct the necessary
reasoning to answer a question of this type.

Question Types
The four major question types in Critical Reasoning are as follows:

(1) Find the Assumption

(2) Draw a Conclusion

(3) Strengthen the Conclusion

(4) Weaken the Conclusion

 Notice that three of the four major question types involve finding the conclusion in order to answer the question,
and the fourth requires us to find the conclusion itself among the answer choices. Clearly, the conclusion is the
most important part of each argument!

Page 30 of 45
VERBAL ABILITY

HSSTS301_4210717

You may also encounter any of seven minor question types:


 Explain an Event or Discrepancy
 Analyze the Argument Structure
 Evaluate the Conclusion
 Fill in the Blanks
 Resolve a Problem
 Provide an Example
 Mimic the Argument
 Classify Statements as True/False based on Content

How to Approach Critical Reasoning Questions?


1. Read the question stem first, and determine what type of question you are facing.
2. Read the passage critically; analyze the basic components of the argument in light of the question.
3. Formulate a correct answer to the question in your head or on scratch paper.
4. Attack the answer choices until only one remains.

Statements and Inferences

Directions for Q1 and Q2: Select the correct alternative from the given choices.

1. Statement:
Country Y has sought the help of country X, to catch the notorious don. Because of this request, country X is
caught between the devil and the deep sea.
Which of the following can be inferred from the above statement?
(a) Country X does not want to antagonize Country Y or the don.
(b) The don is in country X.
(c) Country X can find out the whereabouts of the don.
(d) The don is involved in illegal activities.
2. Statement:
If we throw ethics to the winds and dance with the powerful, we must remember that we may step on the devil’s
tail. So let’s dance with caution.
Which of the following can be inferred from the above statement?
(a) Dancing with the powerful is necessary.
(b) One has to throw ethics to the winds if one wants to dance with the devil.
(c) The powerful are devils.
(d) None of these
Statements and Course of Action
Directions for Q3 and Q4: Select the correct alternative from the given choices.

3. Now-a-days, many sky scrapers are coming up, leaving no place for children to play. The children are forced to
play on the roads, sometimes causing accidents. Some parents do not allow their children to play, leading to lack
of physical exercise.

Page 31 of 45
VERBAL ABILITY

HSSTS301_4210717

Which of the following would be the most appropriate course of action to solve the problem?
(a) Parents should set up a gym in the house for the physical exercise of the children.
(b) The authorities should not allow building of skyscrapers so that the children will get a playground at
convenient distances.
(c) Children should play in school playgrounds.
(d) None of these
4. Software companies are setting up their offices in the city. Many people from various states are coming to the
city for the job. As the number of people in the city is increasing at a rapid pace, the people are facing problems
in finding residential dwellings. The rentals have doubled in one year in the city, despite having rent control acts.
Very few houses or hostels are available at affordable rents.
Which of the following would be the most appropriate course of action to solve the problem?
(a) Do not allow the software companies to enter the city.
(b) The authorities should enact one more rent control act.
(c) Authorities should build new buildings.
(d) None of these

Statements and Conclusions

Directions for Q5 and Q6: In each of the following questions a statement is given followed by two conclusions
I and II. Mark your answer as:
(a) If only conclusion I follows. (b) If only conclusion II follows.
(c) If neither I nor II follows. (d) If both I and II follow.

5. Statement:
The constitutional amendment carried out in the monsoon session of Parliament prohibits child labour in any
organization.
Conclusions:
I. All employers in India must abide by the new rule.
II. All the victims of child labour will now enroll in schools.

6. Statement:
Company ABC has an envious track record in manufacturing top quality cameras with the latest innovations,
which ensures that the end user gets excellent pictures even in bad weather conditions.
Conclusions:
I. No other company has got as much recognition as ABC in this sector.
II. Even a layman can take great photographs using the cameras made by ABC.

Directions for Q7 and Q8: In each question below is given a statement followed by possible consequences. Find
which of the consequence is most likely.

7. Statement:
Many houses collapsed and many families were left homeless due to a major earthquake that shook Lathur
district, which lies in a seismically sensitive zone.
Possible consequences:
(A) The Government may build a huge multi-storeyed building to give shelter to those who lost their houses.
(B) The government may provide make-shift arrangements for those who lost their houses.
(C) The people may relocate to other districts.
(a) Only A and B (b) Only B (c) Only A (d) Only B and C

Page 32 of 45
VERBAL ABILITY

HSSTS301_4210717

8. Statement:
The Meteorological Department cautioned the government that heavy rains and floods may occur in the next two
days.
Possible Consequences:
(A) The government may evacuate the people from low lying areas.
(B) The meterological department may make arrangements to supply food to the people in the areas that could
get affected.
(C) The government may collect money from the public to help the people who could get affected by the floods.
(a) All of A, B and C (b) Only A and B (c) Only A and C (d) Only A

Directions for Q9 and Q10: Read the following questions and answer accordingly.
Passage Analysis

9. Prolonged spells of hot, dry weather at the end of the grape-growing season typically reduce a vineyard’s yield,
because the grapes stay relatively small. In years with such weather, wine producers can make only a relatively
small quantity of wine from a given area of vineyards. Nonetheless, in regions where wine producers generally
grow their own grapes, analysts typically expect a long, hot, dry spell late in the growing season to result in
increased revenues for local wine producers.
Which of the following, if true, does most to justify the analysts’ expectation?
(a) The lower a vineyards’s yield, the lesser the labor required to harvest the grapes.
(b) Long, hot, dry spells at the beginning of the grape-growing season are rare, but they can have a devastating
effect on a vineyard’s yield.
(c) Grapes grown for wine production are typically made into wine at or near the vineyard in which they were grown.
(d) Grapes that have matured in hot, dry weather make significantly better wine than ordinary grapes.

10. In the past, most children who went sledding in the winter snow in Verland used wooden sleds with runners and
steering bars. Ten years ago, smooth plastic sleds became popular, they go faster than wooden sleds but are
harder to steer and slow. The concern that plastic sleds are more dangerous is clearly borne out by the fact that
the number of children injured while sledding was much higher last winter than it was 10 years ago.
Which of the following, if true in Verland, most seriously undermines the force of the evidence cited?
(a) A few children still use traditional wooden sleds.
(b) Very few children wear any kind of protective gear, such as helmets, while sledding.
(c) Plastic sleds can be used in a much wider variety of snow conditions than wooden sleds can.
(d) Most sledding injuries occur when a sled collides with a tree, a rock, or another sled.

Strengthening & Weakening An Argument

11. Until now only injectable vaccines against influenza have been available. They have been primarily used by older
adults who are at risk for complications from influenza. A new vaccine administered in a nasal spray form has
proven effective in preventing influenza in children. Since children are significantly more likely than adults to
contract and spread influenza, making the new vaccine widely available for children will greatly reduce the
spread of influenza across the population.

Which of the following, if true, most strengthens the argument?


(a) If a person receives both the nasal spray and the injectable vaccine, they do not interfere with each other.
(b) The new vaccine uses the same mechanism to ward off influenza as injectable vaccines do.
(c) Government subsidies have kept the injectable vaccines affordable for adults.
(d) Many parents would be more inclined to have their children vaccinated against influenza if it did not involve
an injection.

Page 33 of 45
VERBAL ABILITY

HSSTS301_4210717

Directions for Q12: Read the following questions and answer accordingly.
Passage Analysis
Strengthening & Weakening An Argument

12. Motorists in a certain country frequently complain that traffic congestion is much worse now than it was 20
years ago. No real measure of how much traffic congestion there was 20 years ago exists, but the motorists
complaints are almost certainly unwarranted. The country’s highway capacity has tripled in the last twenty
years, thanks to a vigorous highway construction program, whereas the number of automobiles registered in the
country has increased by only 75 percent.
Which of the following, if true, most seriously weakens the argument?
(a) Most automobile travel is local, and the networks of roads and streets in the country’s settled areas have
changed little over the last 20 years.
(b) Gasoline prices are high, and miles traveled per car per year have not changed much over the last 20 years.
(c) The country’s urban centers have well-developed public transit systems that carry most of the people who
commute into those centers.
(d) The average age of automobiles registered in the country is lower now than it was 20 years ago.

Statements and Assumptions

Directions for Q13: In each question below is given a statement followed by several assumptions. Find which of the
assumption is implicit in the statement.

13. Statement:
Radios are fast getting replaced by transistors.
Assumptions:
I. Transistors are superior to radios.
II. Government has banned manufacturing of radios.
(a) Only (I) is implicit (b) Only (II) is implicit
(c) Either (I) or (II) is implicit (d) Neither (I) nor (II) is implicit

Statements and Assumptions

Directions for Q14: In each question below is given a statement followed by several assumptions. Find which of the
assumption is implicit in the statement.

14. Statement:
“Do not allow any candidate to leave the examination hall until the examination is over, except when an
emergency calls for.” - An instruction to the supervisor.
Assumptions:
I. The supervisor himself is not supposed to leave the examination hall unattended.
II. The supervisor has the authority to determine the extent of emergency.
III. The examination is for less than two hours.
(a) Only (I) is implicit (b) None is implicit
(c) Only (II) is implicit (d) Only (I) & (II) are implicit

Page 34 of 45
VERBAL ABILITY

HSSTS301_4210717

Theme Detection
Directions for Q15 and Q16: Read each of the short passages given below and choose the option best representing
the theme of the passage.

15. The TRIPS agreement has, as its Preambular objective, a desire to ensure that measures and procedures to
enforce intellectual property rights do not themselves become barriers to legitimate trade. Further, one of its
objectives is to contribute to the transfer and dissemination of technology. It has, among its principles, the
promotion of public interest in sectors of vital importance to the socioeconomic and technological development
of its members. At the same time the agreement recognizes intellectual property rights as private rights. Finally,
the agreement encourages adjustments aimed at higher levels of protection of intellectual property rights.
(a) The TRIPS agreement is more show and less substance.
(b) The benefits of being a signatory to the TRIPS Agreement.
(c) The objectives of TRIPS Agreement.
(d) The deadlines of TRIPS Agreement.
16. During the last two decades, physicians and mental health professionals have begun to discover the limitations
of Western allopathic medicine. The focus is on pathology and disease and not on prevention. The
destructiveness of so many pharmaceutical and surgical remedies, the separation of physical and emotional
problems and the assumption of an asymmetrical, relationship between an all-powerful physician and a
submissive patient have led clinics and researchers to look for answers in other traditions and cultures.
(a) Western allopathic medicine has failed completely.
(b) People have realized the serious limitations of western allopathic medicine.
(c) The narrow focus of western allopathic practices has forced the medical fraternity to search for alternative
systems of health control.
(d) Traditional medicine will come to play an increasing role in the treatment of patients by assuming an
emotional relationship between the doctor and the patient.

Directions for Q17: Read the given statements and answer according to the questions given.

17. Scientists warn of a global warming, a ‘greenhouse effect’ resulting from increased atmospheric pollutants,
including carbon dioxide from the burning of wood, coal and oil. A coal-industry spokesperson says that the
effect need not cause concern in the near future if, as some scientists believe, the Earth faces another ice age
within the next thousand years since each calamity could cancel out the other.
Which of the following, if true, casts the most serious doubt on the conclusion of the spokesperson?
(a) There is a general cyclical pattern in the recurrence of ice ages on Earth.
(b) The disastrous results of the greenhouse effect have begun to occur and will probably intensify within the
next fifty years.
(c) Trees absorb some of the carbon dioxide in the lower atmosphere and produce oxygen, which is not a pollutant.
(d) Much of the carbon dioxide currently being produced comes not from coal but from the burning of trees
cleared from large areas of tropical rain forest.

Critical Reasoning Exercise


18. Insect Infestations in certain cotton growing regions of the world have caused dramatic increases of cotton on
the world market. Knowing that cotton plants mature quickly, many soybean growers in Ortovia plan to cease
growing soybeans, the price of which has long been stable and to begin raising cotton instead, thereby taking
advantage of the high price of cotton to increase their income significantly over the next several years.
Which of the following, if true, most calls into question the reasoning on which the plan is based?

Page 35 of 45
VERBAL ABILITY

HSSTS301_4210717

(a) The cost of raising soybeans has increased significantly over the past several years and is expected to
continue to climb.
(b) Tests of a newly developed, inexpensive pesticide have shown it to be both environmentally safe and effective
against the insects that have infected the cotton crops.
(c) In the past several years, there has been no sharp increase in the demand for cotton, and for goods made out
of cotton.
(d) Many consumers consider cotton cloth a necessity rather than a luxury and would be willing to pay
significantly higher prices for cotton goods than they are currently paying.
(e) The species of insect that has infested the cotton plants has never been known to infest soybean plants

19. Finding of a survey of Systems magazine subscribers: Thirty percent of all merchandise orders placed by
subscribers in response to advertisements in the magazine last year were placed by subscribers under age
thirty-five.
Finding of a survey of advertisers in Systems magazine: Most of the merchandise orders placed in response to
advertisements in Systems last year were placed by people under age thirty-five.
For both of the findings to be accurate, which of the following must be true?
(a) More subscribers to Systems who have never ordered merchandise in response to advertisements in the
magazine are age thirty-five or over than are under age thirty-five.
(b) Among subscribers to Systems, the proportion who are under age thirty-five was considerably lower last year
than it is now.
(c) Most merchandise orders placed in response to advertisements in Systems last year were placed by Systems
subscribers over age thirty-five.
(d) Last year, the average dollar amount of merchandise orders placed was less for subscribers under age thirty-
five than for those aged thirty-five or over.
(e) Last year many people who placed orders for merchandise in response to advertisements in Systems were
not subscribers to the magazine.

20. Which of the following most logically completes the passage?


Concerned about the well-being of its elderly citizens, the government of Runagia decided two years ago to
increase by 20 percent the government-provided pension paid to all Runagians over 65. Inflation in the
intervening period has been negligible, and the increase has been duly received by all eligible Runagians.
Nevertheless, many of them are no better off financially than they were before the increase, in large part because
______________.
(a) they rely entirely on the government pension for their income.
(b) runagian banks are so inefficient that it can take up to three weeks to cash a pension check.
(c) they buy goods whose prices tend to rise especially fast in times of inflation.
(d) the pension was increased when the number of elderly Runagians below the poverty level reached an all-time
high.
(e) in Runagia children typically supplement the income of elderly parents, but only by enough to provide them
with a comfortable living

Answer Key

1. Ans: [d]
Choice (a): From the phrase “caught between the devil and the deep sea”, it can be concluded that country X
does not want to antagonize either party.
Choice (b): The statement does not give any clue, regarding the whereabouts of the don.
Hence, (a) is a conclusion.

Page 36 of 45
VERBAL ABILITY

HSSTS301_4210717

Choice (c): As FBI has sought the help of country X, it is the assumption of FBI that country X can find out the
whereabouts of the don.
Hence, (c) is an assumption.
Choice (d): As it is stated that the don is notorious, it can be inferred that he is involved in illegal activities.
Hence, (d) is an inference.

2. Ans: [d]
Choice (a): As it is asked, to dance with caution, it can be concluded that the situation is such that dancing with
the powerful is necessary. Hence, (a) is a conclusion.
Choice (b): The last sentence ‘dance with caution’ indicates, that it is not necessary to throw ethics to the wind,
if one wants to dance with the devil.
Choice (c): The last sentence ‘dance with the powerful’ and ‘stepping on the devil’s tail’ is understood that the
author assumes that the powerful are devils. Hence, (c) is an assumption.
Choice (d) is the answer.

3. Ans: [d]
Here the problem is the lack of playground. (a) is not a solution for the given problem. (b) is a negative course of
action. Children cannot remain in school for long. Hence, (c) is not feasible.
Choice (d) is the answer.

4. Ans: [d]
Here the problem is that the infrastructure has not developed to match the inflow of people. So (d) is a proper
course of action. (a) is a negative course of action while (b) assumes the existing rules are not sufficient and (c) is
not a practical solution.

5. Ans: [a]
The statement tells us that as a result of an amendment in the constitution children cannot be employed by any
organization. Since it is a constitutional amendment, it follow that all organizations should abide by it. However,
from the statement we cannot deduce what these children would do once they are out of their jobs, hence
conclusion II does not follow. However, I follows, therefore (a) is the answer.

6. Ans: [c]
The statement does not indicate that ABC is the only company that has an envious track record. Hence,
conclusion I does not follow. Also the statement does not describe the expertise required to handle such a
camera. Hence, conclusion II does not follow. Therefore, the answer is choice (c).

7. Ans: [b]
The place is located in a seismically sensitive area. In such places, a multi storeyed building is fraught with risk.
Hence (a) is not a possible consequence.
The government takes immediate relief measures as a first step. One among such relief measures is a private
make step. One among such relief measures is a private make shift arrangements. Hence (b), is a possible
consequence. Relocating the people to other district is not a practical solution. As it leads to other problems like
lose of income source etc. Hence (c), is not a possible consequence.
Choice (b) is the answer.

8. Ans: [d]
As a precaution, the government may evacuate the people from low-lying areas. Hence, (a) is a possible
consequence. It is not the duty of the Meteorological Department to take up rescue operations. Hence, (b) is not a
possible consequence.
The Government maintains a separate fund to deal with natural calamities. Moreover, no damage had taken
place so far. Hence (c) is not a possible consequence.
Choice (d) is the answer.

Page 37 of 45
VERBAL ABILITY

HSSTS301_4210717

9. Ans: [d]
Choice (a): Lower labor costs mean less expenditure for the winemakers; this does not explain how revenues
would increase.
Choice (b): This statement about low yields does not explain an increase in revenues.
Choice (c): The proximity of production to the vineyard is irrelevant to the question of how hot the dry weather
can be responsible for decreased yield and increased revenues.
Choice (d): Correct. This statement properly provides the explanation that the weather conditions will lead to
better wines. With better wines typically commanding higher prices, the winemakers will gain the increased
revenues that the analysts anticipate. The correct answer is option (d).

10. Ans: [c]


Choice (a): The limited use of some wooden sleds does not weaken the argument.
Choice (b): The absence of protective gear would affect accidents with both kinds of sleds.
Choice (c): This statement weakens the argument by providing an alternate explanation for the increased
accidents.
Choice (d): This statement is true of accidents with both the kinds of sleds. The correct answer is option (c).
11. Ans: [d]

12. Ans: [a]


Choice (a): Correct. This statement properly identifies a weakness in the argument: the response to the road
complaint addresses a different subject, highway capacity, not the issue of traffic congestion encountered by
most motorists.
Choice (b): If high gas prices actually prevented motorists from driving and if motorists’ driving habits were the
same as they were twenty years ago, then these points should strengthen the argument that there is no basis for
their complaints.
Choice (c): The number of commuters who use public transit does not affect the argument that the motorists’
complaints have no basis.
Choice (d): The age of registered cars is irrelevant to the argument. The correct answer is option (a).

13. Ans: [a]


Assumption (I) is implicit since superiority of transistors over radios must be the decisive factor for the
replacement of radios by transistors. Assumption (II) is not implicit since the statement does not, at any stage
imply the banning on the manufacturing of radios by the government.

14. Ans: [d]


Assumption (I) is implicit since it is the supervisor who is going to be in-charge at the examination hall.
Assumption (II) is implicit since the extent of emergency requirement would naturally have to be decided on the
spot (and cannot be predefined). Assumption (III) is not implicit since nothing can be implied about the duration
of the examination.

15. Ans: [c]


The passage enumerates the various objectives of TRIPs Agreement as: ‘ensuring that measures and procedures
to enforce intellectual property rights do not themselves become barriers.’ ‘Contribution to transfer and
dissemination of technology,’ ‘promotion of public interest in sectors of vital importance,’ ‘encouraging
adjustments aimed at higher levels of protection of intellectual property rights’. Option (c) is correct as it best
suits the theme of the passage. Option (a) is incorrect as it is not mentioned anywhere in the passage. Option (b)
is incorrect as it partially represents the theme of the passage. Option (d) is incorrect as there has been no
mention of deadlines in the passage.

Page 38 of 45
VERBAL ABILITY

HSSTS301_4210717

16. Ans: [c]


The passage itself talks of the limitations of Western allopathic medicine,’ ‘focus on pathology and disease rather
than on prevention,’ and ‘the separation of physical and emotional problems.’ Option (c) is correct as it best suits
the theme of the passage. Option (a) is incorrect as this has not been mentioned anywhere in the passage. Option
(b) is incorrect as it is partially correct in the given context. Option (d) is incorrect as assuming an emotional
relationship between the doctor and the patient will not be enough.

17. Ans: [b]


As per the spokesperson’s conclusion, the earth faces another ice age within the next thousand years and this
could cancel out the effect of global warming. The ice age is expected within the next thousand years but we
cannot wait for the next thousand years as the green house affect may have begun. What if it causes damage
within next 30 or 50 years? This argument would weaken the spokeperson’s conclusion. Option (b) highlights
this and is the correct answer.

18. Ans: [b]


Tests of a newly developed, inexpensive pesticide have shown it to be both environmentally safe and effective
against the insects that have infected the cotton crops.

19. Ans: [e]


Last year many people who placed orders for merchandise in response to advertisements in Systems were not
subscribers to the magazine.

20. Ans: [e]


In Runagia children typically supplement the income of elderly parents, but only by enough to provide them with
a comfortable living

SESSION – 8
CONSOLIDATED LEARNING
Directions for Q1 to Q5: Read the following short passages and pick the best option for the questions that follow.

Comprehensive lifestyle changes including a better diet and more exercise can lead not only to a better physique, but
also to swift and dramatic changes at the genetic level, U.S. researchers said on Monday. In a small study, the
researchers tracked 30 men with low-risk prostate cancer who decided against conventional medical treatment such
as surgery and radiation or hormone therapy.

The men underwent three months of major lifestyle changes, including eating a diet rich in fruits, vegetables, whole
grains, legumes and soy products, moderate exercise such as walking for half an hour a day, and an hour of daily
stress management methods such as meditation. As expected, they lost weight, lowered their blood pressure and saw
other health improvements. But the researchers found more profound changes when they compared prostate biopsies
taken before and after the lifestyle changes. After the three months, the men had changes in activity in about
500 genes -- including 48 that were turned on and 453 genes that were turned off. The activity of disease-preventing
genes increased while a number of disease-promoting genes, including those involved in prostate cancer and breast
cancer, shut down, according to the study published in the journal Proceedings of the National Academy of Sciences.

The research was led by Dr. Dean Ornish, head of the Preventive Medicine Research Institute in Sausalito, California,
and a well-known author advocating lifestyle changes to improve health. "It's an exciting finding because so often
people say, 'Oh, it's all in my genes, what can I do?' Well, it turns out you may be able to do a lot," Ornish, who is also
affiliated with the University of California, San Francisco, said in a telephone interview. "'In just three months, I can
change hundreds of my genes simply by changing what I eat and how I live?' That's pretty exciting," Ornish said. "The
implications of our study are not limited to men with prostate cancer."

Page 39 of 45
VERBAL ABILITY

HSSTS301_4210717

1. The article basically states that a healthy lifestyle


(a) can even alter your genes for the better.
(b) has no effect on your genes.
(c) is good for the environment.

2. Apart from eating healthy food and exercising, the men in the study
(a) were put under a lot of stress.
(b) were taught stress management methods.
(c) were given a lot of meat.

3. In total, how many genes changed as a result of the healthy lifestyle?


(a) 48 (b) 453 (c) 500

4. What happened to some of the disease-causing genes?


(a) Their activity increased
(b) Their activity decreased
(c) There was very little change in their activity

5. Which of the following is NOT mentioned in the article? - The men in the study
(a) complained about headaches (b) lost weight
(c) lowered their blood pressure

Directions for Q6 to Q8: Read the following short passages and pick the best option for the questions that follow.

6. Guillemots are birds of Arctic regions. They feed on fish that gather beneath these sheets of floating ice, and they
nest on nearby land. Guillemots need 80 consecutive snow-free days in a year to raise their chicks, so until
average temperatures in the Arctic began to rise recently, the Guillemots’ range was limited to the southernmost
Arctic coast. Therefore, if the warming continues, the Guillemots’ range will probably be enlarged by being
extended northward along the coast.
What of the following, if true, most seriously weakens the argument?
(a) Even if the warming trend continues, there will still be years in which Guillemot chicks are killed by an
unusually early snow.
(b) If the Arctic warming continues, Guillemots’ current predators are likely to succeed in extending their own
range farther north.
(c) Guillemots nest in coastal areas, where temperatures are generally higher than in inland areas.
(d) If the Arctic warming continues, much of the thin ice in the southern Arctic will disappear.

7. Gortland has long been narrowly self-sufficient in both grain and meat. However, as per capita income in
Gortland has risen toward the world average, per capita consumption of meat has also risen towards the world
average, and it takes several pounds of grain to produce one pound of meat. Therefore, per capita income will
continue to rise, whereas domestic grain production will not increase, Gortland will soon have to import either
grain or meat or both.
Which of the following is an assumption on which the argument depends?
(a) The total acreage devoted to grain production in Gortland will soon decrease.
(b) Importing either grain or meat will not result in a significantly higher percentage of Gortlander’s incomes
being spent on food than is currently the case.
(c) The per capita consumption of meat in Gortland is increasing at roughly the same rate across all income levels.
(d) People in Gortland who increase their consumption of meat will not radically decrease their consumption of
grain.

Page 40 of 45
VERBAL ABILITY

HSSTS301_4210717

8. Some economists view the Kennedy-Johnson tax cut of 1964, which radically reduced corporate and individual
taxes, as the impetus for the substantial prosperity enjoyed by the United States in the late 1960’s and early
1970’s.
Which of the following, if true, would most weaken the claim that the tax cut of 1964 was the impetus for
economic prosperity?
(a) Modernized, more productive factories were built in the late 1960’s as a result of funds made available by
the tax cut.
(b) Improved economic conditions in Western Europe and Japan resulted in substantially increased demand for
United States manufactured goods in the late 1960’s.
(c) The tax cut of 1964 contained regulations concerning tax shelters that prompted investors to transfer their
savings to more economically productive investments.
(d) Personal income after taxes rose in the years following 1964.

Directions for Q9: Read each of the short passages given below and choose the option best representing the theme of
the passage.

9. The hydrosphere has strange characteristics because water has properties unlike those of any other liquid. One
anomaly is that on freezing water expands, whereas most liquids contract on cooling. For this reason, ice floats
on water bodies instead of sinking to the bottom. If the ice sank, the hydrosphere would soon be frozen, except
for a thin layer of surface melt water during summer. Then, all aquatic life would be destroyed and the
interchange of warm and cold currents, which moderates climate, would stop.
(a) One of the properties of ice, i.e., floating on water, plays the main role in moderating the climate.
(b) The anomaly of water expanding on cooling is a boon for the moderation of climate and for all aquatic life.
(c) The strange characteristics of the hydrosphere make the floating of ice possible on water bodies.
(d) Ice does not sink in water; this property of ice has ensured the preservation of all aquatic life.

10. One theory of the Moon’s origin is that the Earth, early in its development, was a rapidly rotating body of molten
rock in which most of the iron had settled to the core; some of this fluid was cast off from the surface of the
spinning mass and later solidified to form the Moon.
Which of the following statement can be most directly inferred from the above?
(a) The Moon is the only sizable heavenly body in orbit around the Earth.
(b) The Moon has proportionally less iron at its core than the Earth does.
(c) The surface of the Moon solidified after the surface of the Earth did.
(d) Most of the fluid cast off from the Earth was dispersed into outer space.

11. Hindustan Lever Limited (HLL), a subsidiary of the giant corporation Unilever, is the largest consumer company
in India. Following its merger with Kwality, it has become the largest company in the ice-cream industry. HLL’s
mergers and acquisitions route has provided it access to new high growth business segments like ice-creams,
cosmetics, etc. HLL enjoys the reputation of being one of the most investor-friendly companies with an excellent
dividend track record and a high average payout ratio.
Which of these can be deduced from the passage?
(a) HLL’s growth is organic and hence it is dominant in the Indian market.
(b) HLL is rethinking its survival strategy, shunning its complacency, in the wake of the opening of the economy.
(c) HLL has merged with Kwality to emerge as India’s largest ice-cream company.
(d) All of the above

Page 41 of 45
VERBAL ABILITY

HSSTS301_4210717

12. A major impediment to wide acceptance of electric vehicles even on the part of people who use their cars almost
exclusively for commuting is the inability to use electric vehicles for occasional extended trips. In an attempt to
make purchasing electric vehicles more attractive to commuters, one electric vehicle producer is planning to offer
customers three days free rental of a conventional car for every 1,000 miles that they drive their electric vehicle.
Which of the following, if true, most threatens the plan’s prospects for success?
(a) Many electric vehicles that are used for commercial purposes are not needed for extended trips.
(b) Because a majority of commuters drive at least 100 miles a week, the cost to the producer of making good
the offer would add considerably to the already high price of electric vehicles.
(c) The relatively long time it takes to recharge the battery of an electric vehicle can easily be fitted into the
regular patterns of car use characteristics of commuters.
(d) Although electric vehicles are essentially emission-free in actual use, generating the electricity necessary for
charging as electric vehicle’s battery can burden the environment.

13. The increase in the number of newspaper articles exposed as fabrications serves to bolster the contention that
publishers are more interested in boosting circulation that in printing the truth. Even minor publications have
staff to check such obvious fraud.
The argument above assumes that
(a) newspaper stories exposed as fabrications are a recent phenomenon.
(b) everything a newspaper prints must be factually verifiable.
(c) fact checking is more comprehensive for minor publications than for major ones.
(d) the publishers of newspapers are the people who decide what to print in their newspapers.

14. A common defense of sport hunting is that it serves a vital wildlife-management function, without which
countless animals would succumb to starvation and disease. This defense leads to the overly hasty conclusion
that sport hunting produces a healthier population of animals.
Which of the following, if true, best supports the author’s claim that sport hunting does not necessarily produce a
healthier population of animals?
(a) For many economically depressed families, hunting helps keep food on the table.
(b) Wildlife species encroach on farm crops when other food supplies become scarce.
(c) Over population of a species causes both strong and weak animals in a population.
(d) Sport hunters tend to pursue the biggest and healthiest animals in a population.

Directions for Q15 and 16: Each question has a number of sentences which, when properly sequenced, form a
coherent paragraph. Each sentence is labeled with a letter. Choose the most logical order of sentences from among the
given choices to construct a coherent paragraph.

15. A. So it is not always a blessing to have house servants.


B. Sometimes he may even teach bad habits to our children.
C. Or he may be lazy.
D. He may be dishonest.
E. A servant might not always be a trustworthy fellow.
(a) ABCDE (b) DEABC (c) CDEAB (d) EDCBA

16. A. When I speak of intelligence, I include two things.


B. There is one great defect in traditional morality.
C. The other is receptivity to knowledge.
D. One is actual knowledge.
E. It puts low store by intelligence.
(a) BEADC (b) CDAEB (c) AEBCD (d) EBCDA

Page 42 of 45
VERBAL ABILITY

HSSTS301_4210717

Directions for Q17 and 18: Sentences given in each question, when properly sequenced, form a coherent paragraph.
The first and last sentences are 1 and 6, and the four in between are labeled A, B, C and D. Choose the most logical order
of these four sentences from among the five given choices to construct a coherent paragraph from sentences 1 to 6.

17. 1. The smaller mobile phones get; the more functions they seem to acquire.
A. Beginning early next year, TU Media, a consortium of SK Telecom and other Korean companies, will be
beaming television programmes via satellite to special Digital Multimedia Broadcasting phones.
B. The phones, made by Samsung, have 5.5 centimetre screens and cost $700.
C. Now a South Korean firm is rolling out mobiles that double as televisions–and high-definition ones, at that.
D. Based on early tests, executives at TU Media think that the new service will improve on SK’s existing
cellular-based television phone service, which has 3 million subscribers but is often interrupted due to poor
transmission.
6. The biggest problem, though, is coming up with programming.
(a) BDAC (b) BADC (c) CABD (d) CBAD (e) CADB

18. In the late eighteenth- and early nineteenth-century British society, on top was the royalty, followed by the
aristocracy, then by the gentry, and then by the lower classes, who made up the vast majority of the population.
1. The social status of aristocrats was a formal and settled matter, because aristocrats had official titles.
B. Although the gentry, or upper middle class, possessed servants and often large estates, they held a
nonetheless fragile social position.
C. Members of the gentry, however, held no titles, and their status was thus subject to change.
D. A man might see himself as a gentleman but find, to his embarrassment, that his neighbours did not share this
view.
6. A discussion of whether or not a man was really a gentleman would consider such questions as how much
land he owned, how many tenants and servants he had, how he spoke, whether he kept horses and a
carriage, and whether his money came from land or “trade”—gentlemen scorned banking and commercial
activities.
(a) ABCD (b) BACD (c) ACBD (d) DBCA

Directions for Q19 and Q20: The first and the last parts of the sentence are marked 1 and 6. The rest of the sentence
is split into four parts and marked-P, Q, R and S. These four parts are not given in their proper order. Read the
sentences and find out which of the five combinations is correct.

19. 1. The need of administration


P. which could bring together
Q. language of the rulers should be the one li
R. apart from those of liberal education
S. render it necessary that English, as the
6. linguistic areas.
(a) RSQP (b) SRQP (c) PQRS (d) QSPR

20. 1. Economic domination


P. anger and Q. a different language
R. of persons speaking S. often causes
6. a sense of revolt.
(a) PSRQ (b) QRPS (c) RQPS (d) RQSP

Page 43 of 45
VERBAL ABILITY

HSSTS301_4210717

SOLUTION

1. Ans: [a] 2. Ans: [b] 3. Ans: [c] 4. Ans: [b] 5. Ans: [a]

6. Ans: [d]
Choice (a): An exceptional year is not an argument against an enlarged range because an usually early snow
could happen in the southern arctic as well.
Choice (b): If their current predators also migrate northward, then the guillemots’ situation has not changed, so
this is not an argument against their enlarged range.
Choice (c): The argument suggests that they will move not inland, but northward along the coast.
Choice (d): Correct. This statement properly identifies a factor that weakens the argument: the guillemots’
move northward would not enlarge their range if they lost their food source, fish found under thin ice, in
southern Arctic. The correct answer is option (d).

7. Ans: [d]
Choice (a): The argument makes no assumption about the acreage devoted to grain; it assumes only that the
demand for grain will rise.
Choice (b): The argument does not discuss the percentage of their income that Gortlanders spend on food, so an
assumption about this topic is not needed.
Choice (c): The argument involves only meat consumption in general, not its distribution by income level.
Choice (d): Correct. This statement properly identifies the assumption that there will be no great decrease in
grain consumption. The correct answer is option (d).

8. Ans: [b]
Choice (a): The modern, more productive factories made possible by the tax cut offer a reason in support of the
conclusion, not against it.
Choice (b): Correct. This statement properly identifies a factor that weakens the argument by providing an
alternative explanation.
Choice (c): The economically productive investments made possible by the tax cut provide an example that
supports, rather than weakens, the conclusion.
Choice (d): The rise in personal income after 1964 suggests that the tax cut of 1964 was responsible for this
increase. The correct answer is option (b).

9. Ans: [b]
The passage says that water, unlike any other liquid, expands on cooling. If the ice sank, all the water would
freeze. Consequently, all the aquatic life would be destroyed and interchange of warm and cold water currents
would stop. The theme of the passage is this very anomaly of water and not the properties of ice, which
preserves all aquatic life and moderates climate through interchange of water currents. Option (b) is correct as it
best represents the theme of the passage.

10. Ans: [b]


Option (b) can be best supported by the theory, stated in the question, Option (b) says that the moon has
proportionally less iron at its core than the earth does. According to the argument, most of the iron had settled to
the core of the earth during rotation, early in its developmental period. Since, most of the iron had settled,
comparatively less amount of iron was cast off and moon thus has proportionally less iron than earth does.
Option (a) and (d) are absurd ones and cannot, thus, be concluded. Option (c) is mentioned in the passage, but is
not the conclusion.

Page 44 of 45
VERBAL ABILITY

HSSTS301_4210717

11. Ans: [c]


Only option (c) can be directly inferred as the passage says “Following its merger with Kwality, it has become the
largest company in the ice-cream industry”. The other three options are irrelevant.

12. Ans: [b]


Option (b) is the best answer because if the plan would add considerably to the price of the electric vehicle, then
it in effect replaces one obstacle to buying an electric vehicle with another. Option (a) is incorrect because the
producer’s plan is focused on commuters, so the way some electric vehicles are used for commercial purposes is
of no relevance to the plan. Option (c) poses no threat to the plan. Option (d) presents both an advantage and a
disadvantage of using an electric vehicle, but even the disadvantage does not threaten the plan’s prospects of
making electric vehicles more attractive to commuters than they currently are.
13. Ans: [d]
Evidence: more newspaper articles exposed as fabrications. Conclusions: Publishers want to increase circulation.
Publishers want not increase circulations, not print the truth. This makes sense only if we assume option (d), that
publishers decide what to print. If option (d) weren’t true and this decision were up to someone else, the argument
would fall apart. Since the argument claims only an increase in the number of fabricated articles that are exposed, it’s
not necessary that this is a recent phenomenon, so option (a) is not assumed. Option (b) goes too far –it’s not
necessary that every article be factually verifiable in order for there to have been an increase in fabrications. As for
option (c), the author’s claim that “even minor publications” have fact checkers is meant to emphasis that the
publications know they’re not printing the truth, not that minor ones are better at fact checking than major ones.
14. Ans: [d]
The author’s claim and the claim of the passage do not concur with each another. So, the option sought should be
such that it negates the claim of the passage that sport hunting produces the healthiest population of animals.
Option (d) says that sport hunters tend to pursue the biggest and healthiest animals are chased and killed by the
hunters, you have a weaker population of animals remaining. Thus, it shows that sport hunting produces a
weaker population of animals which negates the claim of the passage. (d) is thus the right answer.
15. Ans: [d] 16. Ans: [a] 17. Ans: [c] 18. Ans: [b] 19. Ans: [a]

20. Ans: [d]

Page 45 of 45

You might also like